SlideShare una empresa de Scribd logo
1 de 58
Descargar para leer sin conexión
atic

as

CAP´
ITULO 7

eM

atem

SEMEJANZA

ept

o. d

´
INTRODUCCION

7.1.

a, D

Definici´n 1.
o
a. Raz´n: se llama raz´n, al cociente de dos cantidades,
o
o
expresadas en la misma magnitud, por ejemplo a .
b

dad

de

An
tio

qui

b. Proporci´n: se llama proporci´n a la igualdad de dos razones. Por
o
o
c
e
e
ejemplo a = d , a los t´rminos a y d se les llama extremos y los t´rminos
b
¯ ¯
b y c se les llama medios, al t´rmino d se le llama cuarta proporcional
e
¯ ¯
¯
entre a, b y c en este orden.
¯ ¯ ¯
En algunos textos de geometr´ se utiliza la notaci´n de proporci´n as´
ıa
o
o
ı:
a : b = c : d que se lee “a es a b como c es a d”
¯
¯
¯
¯

ersi

Propiedades de las proporciones:
a
b

=

c
d

entonces a · d = b · c

2. Si

a
b

=

c
d

y

3. Si

a
b

=

c
d

entonces

b
a

4. Si

a
b

=

c
d

entonces

a±b
b

5. Si

a
b

=

c
d

a
b

=

c
e

Un
iv

1. Si

entonces d = e

entonces

=

d
c

=

a+b
c+d

o

a
c

c±d
d

=

=

b
d

o

a±b
a

a−b
c−d

o

o

d
b

=

a+b
a−b

201

=

c
a

c±d
c

=

c+d
c−d
CAP´
ITULO 7. SEMEJANZA, JAIME ESCOBAR A.

202
6. Si

a1
b1

=

a2
b2

=

a3
b3

= ... =

an
bn

entonces

a2
a3
an
a1 + a 2
a1 + a 2 + . . . + a n
a1
=
=
= ... =
=
= ... =
b1
b2
b3
bn
b1 + b 2
b1 + b 2 + . . . + b n

atic

7.2.

as

b
7. Si b es una magnitud tal que a = d , entonces decimos que b es media
b
¯
¯
proporcional entre a y d o lo que es lo mismo: b es media proporcional
¯ ¯2
¯
entre a y d si y solo si b = a · d.
¯ ¯

PARALELISMO Y PROPORCIONALIDAD

atem

←→

B

o. d

P

 

A

eM

o
Definici´n 2.
o
1. Un punto P ∈AB divide al segmento AB en una raz´n
PA
r si P B = r.
Si r = 1 entonces P es el punto medio de AB.
 

 

a, D

ept

Figura 1.

←→

←→

An
tio

qui

2. Sean AB y CD y sean X ∈AB y Y ∈CD, decimos que X e Y dividen
a AB y CD en segmentos proporcionales si

A

X

¡

Y

dad

¡

D

¡

¡

Un
iv

ersi

¡

C

B
¡

de

XA
YC
=
XB
YD

Figura 2.

El siguiente Lema, llamado el Teorema fundamental del paralelismo es en
realidad una generalizaci´n del Teorema de la paralela media.
o
7.2. PARALELISMO Y PROPORCIONALIDAD

203

Lema 1 (Teorema fundamental del paralelismo).
Si tres o m´s rectas paralelas determinan segmentos congruentes en una
a
secante entonces determinan segmentos congruentes sobre cualquier otra
secante.

E

F

m

C

eM

E’
G

n

o. d

F’

H

a, D
b

An
tio

Figura 3.

r

qui

G’

ept

D
a

atic

B

l

atem

A

as

Demostraci´n. (Ver Figura 3.)
o

←→

de

Sean l, m, n, r cuatro rectas paralelas y a una secante que corta a estas
paralelas en A, B, C, D tales que AB ∼ BC ∼ CD. Sea b otra secante que
=
=
corta a las paralelas en E, F, G, H. Veamos que EF ∼ F G ∼ GH.
=
=
←→

←→

Un
iv

ersi

dad

Por Playfair, por E, F, G pasan EE , F F , GG paralelas a a, donde
E ∈ m, F ∈ n, G ∈ r.
Por la proposici´n 2., AB ∼ EE , BC ∼ F F y CD ∼ GG luego EE ∼
o
=
=
=
=
∼ GG y como los angulos E EF ∼ F F G ∼ G GH por correspondientes
FF =
´
=
=
entre paralelas y por la proposici´n n´mero 1, EE F ∼ F F G ∼ GG H y
o u
=
=
por el criterio A-L-A, los siguientes tri´ngulos son congruentes:
a
EE F ∼
=

FF G ∼
=

luego
EF ∼ F G ∼ GH
=
=

GG H,
CAP´
ITULO 7. SEMEJANZA, JAIME ESCOBAR A.

204

Teorema 1.
Dado un n´mero entero n y dado un segmento, existen puntos en el interior
u
del segmento que lo dividen en n segmentos congruentes.

as

An−1 An

l

¢

¢

atem

A2
Figura 4.

atic

¢

A1

¢

A

P2
¢

P1

Pn
B
¢

Pn−1

o. d

eM

Demostraci´n. (Ver Figura 4.)
o
Sea AB un segmento y n un n´mero entero, veamos que existen puntos P
u
que dividen al segmento en n segmentos congruentes. Sea l una semirrecta
←→

a, D

ept

cualesquiera, con origen en A tal que l no est´ contenida en la recta AB.
e
ımedes, existen
Sobre la semirrecta l, por el Axioma de continuidad de Arqu´
puntos A1 , A2 , . . . , An−1 , An tales que

qui

AA1 ∼ A1 A2 ∼ . . . An−1 An
=
=

An
tio

Por Playfair, por A1 , A2 , . . . , An−1 pasan paralelas a An B, las cuales se intersectan con AB en P1 , P2 , . . . , Pn−1 , entonces por el lema anterior

de

AP1 ∼ P1 P2 ∼ . . . Pn−1 B
=
=

Un
iv

ersi

dad

Definici´n 3 (Segmentos conmensurables e inconmensurables). Deo
cimos que un segmento es conmensurable si su medida es un n´mero racional
u
y decimos que un segmento es inconmensurable si su medida es un n´mero
u
irracional.
Teorema 2 (Teorema de Tales).
Si tres o m´s paralelas cortan a dos o m´s secantes entonces los segmentos
a
a
que determinan en ellas son proporcionales.
Demostraci´n. (Ver Figura 5.)
o
←→

←→

←→

Sean AD, BE y CF rectas paralelas que cortan las secantes a, b en los puntos
7.2. PARALELISMO Y PROPORCIONALIDAD

A

205

D
D1

A1

D2

A2

D3

A3

An−1

Dn−1

B

E

as

E1

B1

Em

Bm

F Em+1

a

eM

b

atem

C
Bm+1

atic

E2

B2

ept

o. d

Figura 5.

de

An
tio

qui

a, D

A, D, B, E, C, F respectivamente.
AB
EF
Veamos que BC = DE o equivalentemente BC = DE
EF
AB
BC
EF
Llamemos x = AB e y = DE y veamos que x = y
Sea n un n´mero entero cualesquiera, entonces por el Teorema 1., existen
u
puntos A1 , A2 , . . . , An−1 que dividen al segmento AB en n segmentos congruentes:
AA1 ∼ A1 A2 ∼ A2 A3 ∼ . . . ∼ An−1 B,
AB = nAA1 .
=
=
=
=
←→

ersi

dad

Por Playfair, por A1 , A2 , . . . , An−1 pasan rectas paralelas a AD que cortan
a b en D1 , D2 , . . . , Dn−1 , luego por el Lema 1. (Teorema fundamental del
paralelismo), los segmentos:
DD1 ∼ D1 D2 ∼ D2 D3 ∼ . . . ∼ Dn−1 E,
DE = nDD1 .
=
=
=
=

Un
iv

Por el Axioma de Arqu´
ımedes, existen puntos B1 , B2 , . . . , Bm , Bm+1 en la
−
−
→
BC tales que
BB1 ∼ B1 B2 ∼ B2 B3 ∼ . . . ∼ Bm Bm+1 ∼ AA1 ,
=
=
=
=
=
y C entre B, Bm+1 , por tanto BBm = mAA1 .
Luego,
BBm
mAA1
m
=
=
AB
nAA1
n
CAP´
ITULO 7. SEMEJANZA, JAIME ESCOBAR A.

206

←→

Por Playfair, por B1 , B2 , . . . , Bm , Bm+1 pasan paralelas a AD que cortan a
−
→
EF en los puntos
E1 , E2 , . . . , Em , Em+1
y por el Lema 1. (Teorema fundamental del paralelismo),
EE1 ∼ E1 E2 ∼ E2 E3 ∼ . . . ∼ Em Em+1 ,
=
=
=
=

y F entre E y Em+1 , ya que C esta entre B y Bm+1 luego

o b.) x >

atem

m
n

m
.
n

eM

Dos casos pueden ocurrir: a.) x =
a.) Si x = m , entonces
n

atic

EEm
mDD1
m
=
=
DE
nDD1
n

as

EEm = mDD1 .

ept

o. d

m
mAA1
BBm
BC
=x=
=
=
AB
n
nAA1
AB

BC
m
>
nAA1
n

m
n

entonces x =

BC
AB

>

m
n

o sea que

An
tio

b.) Supongamos que x >

qui

a, D

y por lo tanto BC = BBm y como C y Bm est´n del mismo lado con respecto
a
a B entonces por el axioma de construcci´n de segmento, C ≡ Bm , entonces
o
EF
F ≡ Em , luego m = EEm = DE = y.
n
DE

y mAA1 < BC < (m + 1)AA1

de

y

dad

mDD1 < EF < (m + 1)DD1
por lo tanto

Un
iv

ersi

EF
mDD1
m
EF
=
>
= .
DE
nDD1
nDD1
n
En resumen, hemos demostrado que si x > m entonces y > m .
n
n
De la misma manera se demuestra que si y > m entonces x > m .
n
n
Hasta aqu´ hemos demostrado que para todo n´ mero racional m , si
ı,
u
n
x > m entonces y > m y rec´
ıprocamente, si y > m entonces x > m . En
n
n
n
n
otras palabras, todo n´mero racional a la izquierda de x esta tambi´n a la
u
e
izquierda de y y todo n´mero racional a la izquierda de y esta a la izquierda
u
de x. Todo esto significa que no hay un n´ mero racional entre x e y, ya
u
y=
7.2. PARALELISMO Y PROPORCIONALIDAD

207

que si hubiera un n´mero racional entre x e y entonces estar´ a la izquierda
u
ıa
de uno de ellos y a la derecha del otro, lo cual contradice lo demostrado; por
lo tanto x = y, es decir:
AB
DE
=
BC
EF

as

Corolario 1 (Teorema de Tales en el tri´ngulo). Toda recta paralela a
a
un lado de un tri´ngulo y que corte a los otros dos lados, divide a estos lados
a
en segmentos proporcionales.

eM

atem

atic

A

F

o. d

E

C

a, D
qui

Figura 6.

ept

B

luego

y

=

FA
;
FC

por

EA
FA
=
AE + EB
AF + F C

de

FA + FC
EA + EB
=
EB
FC

EA
EB

An
tio

Lo que afirma este corolario es que si EF ||BC entonces
las propiedades de las fracciones

AB
AC
=
AE
AF

Un
iv

ersi

dad

AB
AC
=
y
EB
FC
esto demuestra el siguiente corolario.

Corolario 2. Dos lados de un tri´ngulo son proporcionales a los segmentos
a
que en ellos determina cualquier recta paralela al tercer lado.
El siguiente teorema es el rec´
ıproco del Corolario 1
Teorema 3 (Rec´
ıproco del Teorema de Tales en el tri´ngulo).
a
Si una recta intercepta dos lados de un tri´ngulo en segmentos propora
cionales entonces la recta es paralela al tercer lado del tri´ngulo.
a
CAP´
ITULO 7. SEMEJANZA, JAIME ESCOBAR A.

208

A

F l
E

F’

C

atem

atic

Figura 7.

as

B

l

AE
AF
=
EB
FC

o. d

l ∩ AB = {E}, l ∩ AC = {F },

eM

Demostraci´n. (Ver Figura 7.)
o
Sea el ABC y l una recta tal que

AF
FC

=

AF
F C

An
tio

luego

qui

AE
AF
=
EB
FC

a, D

ept

Por Playfair, por E pasa l ||BC la cual intercepta a AC en F , entonces por
el Corolario 1 (Teorema de Tales en el tri´ngulo), se tiene:
a

y por las propiedades de las fracciones

FC
AF

=

F C
AF

o sea que

dad

que es lo mismo que

de

F C + AF
F C + AF
=
AF
AF

Un
iv

ersi

AC
AC
=
AF
AF
luego AF = AF y por tanto AF ∼ AF y como F, F est´n del mismo lado
a
=
con respecto a A entonces por el axioma de construcci´n de segmento F ≡ F
o
y por lo tanto EF ||BC.
En forma similar se demuestran los siguientes rec´
ıprocos:

Corolario 3 (Rec´
ıproco del Corolario 2). Si dos lados de un tri´ngulo son
a
proporcionales a los segmentos que en ella determina una recta que intercepta
los dos lados, entonces la recta es paralela al tercer lado del tri´ngulo.
a
7.2. PARALELISMO Y PROPORCIONALIDAD

209

A

E

F
C

AB
AC
=
EB
FC

o
´

o. d

AB
AC
=
AE
AF

ABC (Ver Figura 8.)

eM

Lo que afirma este corolario es que si en el

atem

atic

Figura 8.

as

B

a, D

ept

entonces EF ||BC

An
tio

qui

Teorema 4 (Propiedades m´tricas de la bisectriz de un tri´ngulo).
e
a
La bisectriz de un angulo de un tri´ngulo divide al lado opuesto en segmen´
a
tos proporcionales a los otros dos lados.
Demostraci´n. (Ver Figura 9.)
o
Sea AV bisectriz de A en el ABC con V ∈ IntBC. Veamos que
←→

VB
VC

=

AB
.
AC

Un
iv

ersi

dad

de

Por Playfair, por C pasa l||AV ; sea {D} = l∩ BA, luego por alternos
internos entre paralelas, V AC ∼ ACD y por correspondientes entre par=
∼ ADC, pero como AV es bisectriz por hip´tesis, entonces
alelas, BAV =
o
∼ V AC, luego
BAV =
ADC ∼ ACD
=
y por el teorema del tri´ngulo is´sceles, se tiene que
a
o
por lo tanto
AD ∼ AC.
=
Por el corolario 2 (Teorema de Tales en el tri´ngulo),
a
AB
VB
=
,
VC
AD

ADC es is´sceles y
o
CAP´
ITULO 7. SEMEJANZA, JAIME ESCOBAR A.

210

l
D

o. d

C

ept

V
Figura 9.

a, D

B

eM

atem

atic

as

A

qui

luego

An
tio

AB
VB
=
.
VC
AC

ersi

dad

de

Teorema 5 (Rec´
ıproco del teorema anterior).
Si una recta que pasa por el v´rtice de un tri´ngulo divide al lado opuesto
e
a
en segmentos proporcionales a los otros dos lados, entonces esta recta es
bisectriz del angulo ubicado en el v´rtice por donde pasa la recta.
´
e

Un
iv

Demostraci´n. (Ver Figura 10.)
o

←→

Supongamos que en el ABC se tiene que AV con V ∈ IntBC, tal que
VB
= AB . Veamos que AV es bisectriz de A.
VC
AC
←→

Por Playfair, por C pasa l||AV ; sea {D} = l∩ BA.
AB
Como l||AV , entonces por el corolario 2, V B = AD , pero por hip´tesis
o
VC
AB
VB
=
,
VC
AC
7.2. PARALELISMO Y PROPORCIONALIDAD

211

l
D

o. d

C

ept

V
Figura 10.

a, D

B

eM

atem

atic

as

A

entonces

ersi

dad

de

An
tio

qui

AB
AB
=
AC
AD
y por las propiedades de las fracciones AD = AC o sea que AD ∼ AC,
=
por lo tanto el ADC es is´sceles y por el Teorema del tri´ngulo is´sceles,
o
a
o
ADC ∼ ACD.
=
Por otro lado, por alternos internos entre paralelas, V AC ∼ ACD y por
=
∼ ADC.
correspondientes entre paralelas, BAV =
Luego BAV ∼ V AC, luego AV es bisectriz de A.
=

Un
iv

Teorema 6 (Propiedades m´tricas de la bisectriz exterior de un
e
tri´ngulo).
a
La bisectriz de un angulo exterior de un tri´ngulo, que no sea paralela al lado
´
a
opuesto, divide exteriormente al lado opuesto en segmentos proporcionales
a los otros dos lados.
Demostraci´n. (Ver Figura 11.)
o
Sea AV bisectriz del angulo exterior EAC en el
´

←→

ABC con V ∈BC y
CAP´
ITULO 7. SEMEJANZA, JAIME ESCOBAR A.

212

E
A

l

C

B − C − V . Veamos que

V B
V C

=

V’

atem

Figura 11.

eM

B

atic

as

D

AB
.
AC

←→

qui

a, D

ept

o. d

Por Playfair, por C pasa l||AV ; sea {D} = l∩ BA, luego por alternos
internos entre paralelas, V AC ∼ ACD y por correspondientes entre pa=
∼ ADC, pero como AV es bisectriz por hip´tesis, entonces
ralelas, EAV =
o
∼ V AE, luego
CAV =
ADC ∼ ACD
=
ADC es is´sceles y
o

An
tio

y por el teorema del tri´ngulo is´sceles, se tiene que
a
o
por lo tanto
AD ∼ AC.
=

de

Por el corolario 2 (Teorema de Tales en el tri´ngulo),
a

ersi

luego

dad

AB
V B
=
,
V C
AD

Un
iv

V B
AB
=
.
V C
AC

El rec´
ıproco de este teorema se deja como ejercicio.
Teorema 7 (Rec´
ıproco del Teorema anterior).
Una recta que pase por el v´rtice de un tri´ngulo y divida la prolongaci´n del
e
a
o
lado opuesto en segmentos proporcionales a los otros dos lados del tri´ngulo,
a
es bisectriz del angulo exterior ubicado en este v´rtice.
´
e
7.3. SEMEJANZA DE POL´
IGONOS

213

Definici´n 4 (Divisi´n arm´nica). Si A y B son dos puntos distintos y
o
o
o
←→

C ∈ IntAB y D ∈AB pero D ∈ AB, decimos que C, D dividen arm´nica/
o
mente a AB si
DA
CA
=
CB
DB
A
C
B
D
£

£

£

£

atic

as

Figura 12.

ept

o. d

eM

atem

A los puntos C y D se les llama los conjugados arm´nicos con respecto a
o
A y B.
Los puntos A, B, C, D en este orden, se dice que forman una divisi´n arm´nica.
o
o
Tambi´n, de acuerdo a la definici´n, podemos afirmar que A y B son conjue
o
gados arm´nicos con respecto a CD.
o
Por los teoremas 4 y 6 y por la definici´n de conjugado arm´nico, podemos
o
o
afirmar el siguiente teorema.

qui

a, D

Teorema 8.
La bisectriz de un angulo de un tri´ngulo y la bisectriz del angulo exterior
´
a
´
suplementario, dividen al lado opuesto arm´nicamente.
o

SEMEJANZA DE POL´
IGONOS

dad

7.3.

de

An
tio

Nota: de acuerdo a los teoremas anteriores, el lugar geom´trico de los puntos
e
A tales que la raz´n de las distancias a dos puntos fijos B y C sea una
o
constante k, es una circunferencia de di´metro V V , donde V, V son los
a
o
conjugado arm´nicos de BC con raz´n k.
o

Un
iv

ersi

Definici´n 5 (Pol´
o
ıgonos semejantes). Decimos que dos pol´
ıgonos son semejantes si se puede establecer una correspondencia entre sus lados y sus
angulos de tal manera que:
´
1. Los lados correspondientes son proporcionales. A estos lados tambi´n los
e
llamaremos lados hom´logos. La raz´n r entre los lados hom´logos la llao
o
o
mamos raz´n de semejanza.
o
2. Los angulos correspondientes son congruentes. A los angulos correspon´
´
dientes congruentes, tambi´n se les llama angulos hom´logos.
e
´
o
En particular, para los tri´ngulos tenemos la siguiente definici´n.
a
o
CAP´
ITULO 7. SEMEJANZA, JAIME ESCOBAR A.

214

Definici´n 6 (Tri´ngulos semejantes). Decimos que el ABC es semeo
a
jante al A B C , lo cual denotamos as´ ABC ∼ A B C , si:
ı

as

BC
AC
AB
=
=
(∗)
AB
BC
AC
A ∼ A , B ∼ B , C ∼ C (∗∗)
=
=
=
A

atem

eM

B’

o. d

C

a

a

C’

ept

B

b

c

b

c

atic

A’

a, D

Figura 13.

An
tio

qui

Nota: 1. Con los teoremas que haremos m´s adelante, mostraremos que (*)
a
implica (**) y rec´
ıprocamente, (**) implica (*).
2. Por las propiedades de las fracciones, se puede demostrar que si dos
tri´ngulos son semejantes, entonces sus lados son entre si como sus per´
a
ımetros, es decir, si ABC ∼ A B C entonces

dad

de

a
b
c
p
= = = =r
a
b
c
p

Un
iv

ersi

donde p = a + b + c =per´
ımetro del ABC, p = a + b + c =per´
ımetro del
A B C y r es la raz´n de semejanza.
o
3. La relaci´n de semejanza entre pol´
o
ıgonos es una relaci´n de equivalencia,
o
es decir, es reflexiva, sim´trica y transitiva (Ejercicio).
e
Definici´n 7 (Pol´
o
ıgonos congruentes). Decimos que dos pol´
ıgonos semejantes, son congruentes si tienen sus lados hom´logos congruentes.
o
Teorema 9.
Dos pol´
ıgonos semejantes son congruentes si un lado de uno de ellos es
congruente con su hom´logo.
o
7.3. SEMEJANZA DE POL´
IGONOS

215

A continuaci´n veremos tres criterios de semejanza de tri´ngulos.
o
a
Teorema 10 (Primer criterio de semejanza: Angulo-Angulo (A-A)).
Si dos angulos de un tri´ngulo son congruentes con dos angulos de otro
´
a
´
tri´ngulo, entonces los dos tri´ngulos son semejantes.
a
a
A

atic

as

A’

F

C

B’

C’

ept

o. d

B

E

eM

D

atem

l

a, D

Figura 14.

o sea

Un
iv

AB
AC
=
,
AD
AE

ersi

dad

de

An
tio

qui

Demostraci´n. (Ver Figura 14.) Supongamos que en los tri´ngulos ABC
o
a
∼ B , C ∼ C , entonces por el teorema de la suma
y A B C se tiene que B =
=
de los angulos interiores de un tri´ngulo, A ∼ A .
´
a
=
−
→
Por el axioma de construcci´n de segmento, existe un punto D ∈ AB y
o
−
→
E ∈ AC tales que AD ∼ A B y AE ∼ A C ; unamos D con E, entonces
=
=
por el criterio L-A-L, el ADE ∼ A B C , por lo tanto DE ∼ B C ,
=
=
ADE ∼ B , pero por hip´tesis B ∼ B, por lo tanto ADE ∼ B y por el
o
=
=
=
teorema de alternos internos (Teorema 31), DE||BC y por el corolario 2,

←→

AB
AC
=
AB
AC

(∗)

←→

Por Playfair, por D pasa l|| AC, sea {F } = l∩ BC y por la proposici´n
o
∼ F C y como DE ∼ B C entonces F C ∼ B C ; por otro
n´mero 2, DE =
u
=
=
lado, por el corolario 2,
BC
AB
=
,
AD
FC

o sea

AB
BC
=
AB
BC

(∗∗)
CAP´
ITULO 7. SEMEJANZA, JAIME ESCOBAR A.

216
de (*), (**)

AB
AC
BC
=
=
,
AB
AC
BC
hemos mostrado que los tres pares de angulos son congruentes y los tres pares
´
de lados respectivos son proporcionales, por lo tanto
ABC ∼

ABC

as

Se deja como ejercicio los siguientes corolarios.

atem

atic

Corolario 4 (Paralela a un lado de un tri´ngulo). Una paralela a un
a
lado de un tri´ngulo determina otro tri´ngulo semejante al primero.
a
a

eM

Corolario 5. Si dos tri´ngulos rect´ngulos tienen un par de angulos agudos
a
a
´
respectivamente congruentes, entonces son semejantes.

o. d

Corolario 6. Si dos tri´ngulos tienen sus lados respectivamente paralelos o
a
respectivamente perpendiculares, entonces los dos tri´ngulos son semejantes.
a

a, D

ept

Corolario 7. Las alturas y las bisectrices hom´logas de dos tri´ngulos seo
a
mejantes est´n en la misma raz´n que sus lados hom´logos.
a
o
o

qui

Corolario 8. Dos tri´ngulos is´sceles son semejantes si tienen un par de
a
o
angulos congruentes.
´

An
tio

Corolario 9. Todos los tri´ngulos equil´teros son semejantes.
a
a

ersi

dad

de

Teorema 11 (Segundo criterio de semejanza: P-A-P).
Si un angulo de un tri´ngulo es congruente con otro angulo de otro tri´ngulo
´
a
´
a
y los lados que comprenden al angulo en el primer tri´ngulo son respecti´
a
vamente proporcionales a los lados que comprende al angulo en el segundo
´
tri´ngulo, entonces los dos tri´ngulos son semejantes.
a
a

Un
iv

Demostraci´n. (Ver figura 15.) Tomemos por hip´tesis que A ∼ A y
o
o
=
AB
AC
= A C . Veamos que ABC ∼ A B C .
AB
−
→
−
→
Por el axioma de construcci´n de segmento, existen D ∈ AB y E ∈ AC
o
tales que AD ∼ A B y AE ∼ A C , por lo tanto, por el criterio L-A-L,
=
=
∼ ABC .
ADE =
AB
AC
AC
AB
Por otro lado, como A B = A C entonces AD = AE y por el corolario 3
(rec´
ıproco del corolario 2),
DE||BC
7.3. SEMEJANZA DE POL´
IGONOS

217

A
A’

E

B

C

B’

C’

as

D

atem

ADE ∼

ABC ∼

ABC y por transitividad

eM

Por lo tanto, por el corolario 4,

atic

Figura 15.

o. d

ABC

ept

Corolario 10. Dos tri´ngulos rect´ngulos son semejantes si sus catetos son
a
a
respectivamente proporcionales.

qui

a, D

Corolario 11. Las medianas hom´logas de dos tri´ngulos semejantes, estan
o
a
en la misma raz´n que sus lados hom´logos.
o
o

An
tio

Teorema 12 (Tercer criterio de semejanza:P-P-P).
Si los tres lados de un tri´ngulo son respectivamente proporcionales a los
a
tres lados de otro tri´ngulo, entonces los dos tri´ngulos son semejantes.
a
a

A’

D
B

Un
iv

ersi

dad

de

A

E

C

B’

Figura 16.

C’
218

CAP´
ITULO 7. SEMEJANZA, JAIME ESCOBAR A.

Demostraci´n. (Ver Figura 16.) Tomemos por hip´tesis que
o
o
AB
AC
BC
=
=
AB
AC
BC

(∗)

−
→
−
→
Por el axioma de construcci´n de segmento, existen D ∈ AB y E ∈ AC tales
o
que AD ∼ A B y AE ∼ A C , sustituyendo en (*),
=
=

atic

as

AB
AC
=
AD
AE
y por el corolario 3 (rec´
ıproco del corolario 2),

ADE ∼

eM

ABC, de esta semejanza se

BC
AB
=
AB
DE

o. d

Por lo tanto, por el corolario 4,
concluye que

o sea que

(∗∗),

ept

BC
AB
=
AD
DE

atem

DE||BC

pero por hip´tesis
o

qui

a, D

BC
AB
=
(∗ ∗ ∗)
AB
BC
de (**) y (***) y por las propiedades de las fracciones: DE = B C o sea que

An
tio

DE ∼ B C
=

de

y por lo tanto, por el tercer criterio de congruencia de tri´ngulos L-L-L:
a
∼ A B C y como ADE ∼ ABC, entonces por transitividad,
ADE =
ABC.

dad

ABC ∼

7.4.

Un
iv

ersi

Corolario 12. Si las bases de dos tri´ngulos is´sceles son entre si como sus
a
o
otros lados, entonces los tri´ngulos son semejantes.
a

´
SEMEJANZA EN EL TRIANGULO
´
RECTANGULO

Los resultados de aplicar los conceptos de semejanza al tri´ngulo rect´ngua
a
lo son de mucha importancia, pues obtendremos el teorema de Pit´goras y
a
aplicaciones al tri´ngulo y a los cuadril´teros, a las areas, etc.
a
a
´
´
´
7.4. SEMEJANZA EN EL TRIANGULO RECTANGULO

219

Definici´n 8.
o
a. La proyecci´n ortogonal de un punto exterior a una
o
recta, es el punto de intersecci´n de una recta perpendicular desde el
o
punto a la recta.
b. La proyecci´n ortogonal de un segmento sobre una recta es el segmeno
to determinado por las proyecciones ortogonales de los extremos del
segmento sobre la recta.
B

atic

as

P

A’

l

B’

eM

P’

atem

A
¤

ept

o. d

Figura 17.

An
tio

qui

a, D

En la Figura 17., la proyecci´n ortogonal del punto P sobre la recta l es
o
−→
−
−→
−
el punto P , ya que l ⊥ P P y {P } = l ∩ P P .
La proyecci´n ortogonal del segmento AB sobre la recta l es el segmento
o
A B , donde A y B son las proyecciones ortogonales sobre l de A y B
respectivamente.

dad

de

Teorema 13 (Proporcionalidad en el tri´ngulo rect´ngulo).
a
a
Si en un tri´ngulo rect´ngulo se traza la altura correspondiente a la
a
a
hipotenusa, entonces:

ersi

a. Los dos nuevos tri´ngulos que resultan, son semejantes entre si y sea
mejantes al tri´ngulo original.
a

Un
iv

b. La altura es media proporcional entre los segmentos que ella determina
sobre la hipotenusa.
c. Cada cateto es media proporcional entre la hipotenusa y la proyecci´n
o
del cateto sobre la hipotenusa.

Demostraci´n. (Ver Figura 18.)
o
220

CAP´
ITULO 7. SEMEJANZA, JAIME ESCOBAR A.
C

A

B

atic

as

D
Figura 18.

ADC ∼

ABC,

eM

atem

a. Sabemos por el corolario 5, que si dos tri´ngulos rect´ngulos tienen un
a
a
angulo agudo congruente, entonces los dos tri´ngulos son semejantes,
´
a
por lo tanto
CDB ∼

ABC ∼

CDB

ept

ADC ∼

o. d

y por transitividad

ABC

An
tio

qui

a, D

b. Como ADC ∼ CDB y CAD ∼ DCB y ACD ∼ CBD entonces la
=
=
relaci´n entre los lados hom´logos del ADC con los lados hom´logos
o
o
o
del CDB es
AD
AC
DC
ADC
:
=
=
CDB
CD
CB
DB

de

luego CD2 = AD · DB o sea que CD es media proporcional entre AD
y DB.

ersi

dad

c. Como ADC ∼ ABC y ACD ∼ CBA y el angulo A es com´n,
´
u
=
entonces la relaci´n entre los lados hom´logos del ADC con los lados
o
o
hom´logos del ABC es
o

Un
iv

ADC
:
ABC

AD
AC
DC
=
=
AC
AB
CB

luego AC 2 = AD · AB o sea que AC es media proporcional entre AD
y AB.
Como CDB ∼ ABC, BCD ∼ CAB y el angulo B es com´n, se
´
u
=
demuestra en forma similar que
CB 2 = AB · DB
´
´
7.4. SEMEJANZA EN EL TRIANGULO RECTANGULO

221

o sea que CB es media proporcional entre AB y DB.
Teorema 14 (Teorema de Pit´goras).
a
El cuadrado de la medida de la hipotenusa en un tri´ngulo rect´ngulo es
a
a
igual a la suma de los cuadrados de las medidas de los catetos.

B

eM

D
Figura 19.

o. d

A

atem

atic

as

C

a, D

ept

Demostraci´n. (Ver Figura 19.) Sea ABC un tri´ngulo rect´ngulo en
o
a
a
C y sea CD la altura relativa a la hipotenusa, entonces por la parte c. del
anterior teorema:
CB 2 = AB · DB

An
tio

y sumando estas dos expresiones, tenemos

qui

AC 2 = AD · AB,

de

AC 2 + CB 2 = AD · AB + AB · DB = AB(AD + DB) = AB · AB = AB 2

Un
iv

ersi

dad

Teorema 15 (Rec´
ıproco del teorema de Pit´goras).
a
Si en un tri´ngulo el cuadrado de la medida de un lado es igual a la suma
a
de los cuadrados de las medidas de los otros dos lados, entonces el tri´ngulo
a
es rect´ngulo.
a
Demostraci´n. (Ver Figura 20.) Sea el ABC tal que AC 2 = AB 2 +
o
BC 2 . Veamos que el ABC es rect´ngulo en B. Para ello, construyamos un
a
tri´ngulo A B C rect´ngulo en B , as´ : en una recta l fijo un punto B ,
a
a
ı
por el axioma de construcci´n de segmento, existe un punto C en una de las
o
semirrectas determinadas por B en l, tal que B C ∼ BC; por el teorema de
=
la perpendicular por un punto de una recta, por B pasa m ⊥ l, por el axioma
de construcci´n de segmento, existe un punto A en una de las semirrectas
o
CAP´
ITULO 7. SEMEJANZA, JAIME ESCOBAR A.

222

m
A

A’

l
C

as

B’

C’

atic

B

atem

Figura 20.

ept

A C 2 = A B 2 + B C 2.

A B C es

o. d

eM

determinadas por B en m, tal que B A ∼ BA, por lo tanto el
=
rect´ngulo en B . Por el teorema de Pit´goras
a
a

ABC y

AB = A B ,

BC = B C

de

AC = A C ,

A B C se tiene:

An
tio

En los tri´ngulos
a

qui

A C = AC

a, D

Pero por hip´tesis AC 2 = AB 2 + BC 2 , luego A C 2 = AC 2 y por tanto
o

dad

luego, por el criterio L-L-L, se tiene que

ersi

ABC ∼
=

ABC
ABC es rect´ngulo
a

Un
iv

luego ABC ∼ A B C y como A B C es recto, entonces
=
en B.

7.5.

APLICACIONES DEL TEOREMA DE
´
PITAGORAS

Con los siguientes teoremas se demuestra la ley de cosenos en trigonometr´
ıa.
´
7.5. APLICACIONES DEL TEOREMA DE PITAGORAS

223

Teorema 16 (Ley de cosenos).
a. En un tri´ngulo obtus´ngulo, el cuadrado de la medida del lado opa
a
uesto al angulo obtuso es igual a la suma de los cuadrados de las
´
medidas de los otros dos lados, m´s el doble producto de la medida
a
de uno de estos lados por la proyecci´n del otro sobre ´l.
o
e

atem

atic

as

b. En un tri´ngulo cualquiera, el cuadrado de la medida del lado opuesto
a
al angulo agudo es igual a la suma de los cuadrados de las medidas
´
de los otros dos lados, menos el doble producto de la medida de uno
de estos lados por la proyecci´n del otro sobre ´l.
o
e

eM

A

B

ept

o. d

A

H1

H

a, D

C

qui

B

An
tio

H1
(a)

C

H

(b)

dad

de

Figura 21.

ersi

Demostraci´n. a.) (Ver Figura 21.(a)) Supongamos que en el
o
←→

ABC el

o
angulo ABC es obtuso y sea BH la proyecci´n de AB sobre BC y sea BH1
´

Un
iv

←→

la proyecci´n de BC sobre AB, por el Teorema ??, H − B − C y A − B − H1 ;
o
veamos que
AC 2 = AB 2 + BC 2 + 2 · BC · BH

y AC 2 = AB 2 + BC 2 + 2 · AB · BH1

Demostremos la primera expresi´n, la otra se hace en forma similar.
o
Por el teorema de Pit´goras en el AHB se tiene
a
AB 2 = AH 2 + HB 2

(∗)
CAP´
ITULO 7. SEMEJANZA, JAIME ESCOBAR A.

224

Por el teorema de Pit´goras en el
a

AHC se tiene

AC 2 = AH 2 + HC 2

(∗∗)

restando (**) y (*): AC 2 −AB 2 = HC 2 −HB 2 (∗∗∗), pero como H −B −C,
entonces HC = HB + BC y sustituyendo en (***) y despejando

atic

as

AC 2 = AB 2 + (HB + BC)2 − HB 2
= AB 2 + HB 2 + BC 2 + 2 · HB · BC − HB 2
= AB 2 + BC 2 + 2 · BC · HB

ABC el angulo ABC es
´

atem

b.) (Ver Figura 21.(b)). Supongamos que en el
←→

agudo y sea BH la proyecci´n de AB sobre BC y sea BH1 la proyecci´n de
o
o

eM

←→

BC sobre AB, por el Teorema ??, B − H − C y B − H1 − A; veamos que
y AC 2 = AB 2 + BC 2 − 2 · AB · BH1

o. d

AC 2 = AB 2 + BC 2 − 2 · BC · BH

a, D

ept

Demostremos la primera expresi´n, la otra se hace en forma similar.
o
Por el teorema de Pit´goras en el AHB se tiene
a
(∗)

qui

AB 2 = AH 2 + HB 2

AHC se tiene

An
tio

Por el teorema de Pit´goras en el
a

AC 2 = AH 2 + HC 2

(∗∗)

dad

de

restando (**) y (*): AC 2 −AB 2 = HC 2 −HB 2 (∗∗∗), pero como B −H −C,
entonces HC = BC − HB y sustituyendo en (***) y despejando

Un
iv

ersi

AC 2 = AB 2 + (BC − HB)2 − HB 2
= AB 2 + BC 2 + HB 2 − 2 · BC · HB − HB 2
= AB 2 + BC 2 − 2 · BC · HB
Teorema 17 (Teorema de Stewart).
En el

ABC, D ∈ IntBC. Si BD = m, DC = n, AD = d, entonces
d2 a = b2 m + c2 n − amn
´
7.5. APLICACIONES DEL TEOREMA DE PITAGORAS

225

A

c

b
d

D
m

H
n

a

C

as

B

atem

atic

Figura 22.

Demostraci´n. (Ver Figura 22.) Sea D ∈ BC en el
o
←→

ABC, sea DH la

a, D

ept

o. d

eM

proyecci´n de AD sobre BC; con el ADB pueden suceder tres casos: i. que
o
sea obtuso, ii. que sea recto, iii. que sea agudo.
Mostremos el primer caso, los otros casos son similares.
Como ADB es obtuso, entonces por el Teorema ?? B − D − H y ADC es
agudo y BD + DC = BC; por el teorema anterior (ley de cosenos) en el
ADB y en el ADC:
(∗)

AC 2 = AD2 + DC 2 − 2 · DC · DH

(∗∗)

An
tio

qui

AB 2 = AD2 + BD2 + 2 · BD · DH

multiplicando (*) por DC y (**) por BD y luego sumando:

dad

de

AB 2 · DC + AC 2 · BD = AD 2 · (DC + BD) + BD 2 · DC + DC 2 · BD
= AD2 · BC + BD · DC(DC + BD)
= AD2 · BC + BD · DC · BC

ersi

luego AD2 · BC = AB 2 · DC + AC 2 · BD − BD · DC · BC es decir,

Un
iv

d2 a = b2 m + c2 n − amn.
Teorema 18.
a.) La suma de los cuadrados de las medidas de dos lados de un tri´ngulo
a
es igual a dos veces el cuadrado de la medida de la mediana del tercer lado
m´s la mitad del cuadrado de la medida del tercer lado.
a
b.) La diferencia de los cuadrados de las medidas de dos lados de un tri´ngulo
a
es igual a dos veces el producto de la medida del tercer lado por la proyecci´n
o
de la mediana correspondiente a este lado.
226

CAP´
ITULO 7. SEMEJANZA, JAIME ESCOBAR A.
A

c

b

ma
M

B

H

C

as

a

atem

atic

Figura 23.

eM

Demostraci´n. (Ver Figura 23.) En el ABC, sea M el punto medio de
o
BC, ma la mediana relativa al lado BC y M H la proyecci´n de la mediana
o
←→

qui

a, D

ept

o. d

AM = ma sobre BC, supongamos que AB > AC. Con el angulo AM B
´
pueden suceder tres casos: i. es obtuso, ii. es recto, iii. es agudo.
Tomemos el caso i. y veamos que
a.) c2 + b2 = 2 · m2 + 1 a2
a
2
b.) c2 − b2 = 2 · a · M H.
En efecto, como AM B es obtuso entonces AM C es agudo, luego por el
teorema de la ley de cosenos en el AM B y en el AM C:
(∗)

AC 2 = AM 2 + M C 2 − 2 · M C · M H

(∗∗)

de

An
tio

AB 2 = AM 2 + BM 2 + 2 · BM · M H

ersi

dad

sumando (*) y (**) y teniendo en cuenta que M es punto medio, o sea que
M B = M C, entonces

Un
iv

AB 2 + AC 2 = 2 · AM 2 + BM 2 + M C 2
BC 2
BC 2
= 2 · AM 2 +
+
2
2
BC 2
1
= 2 · AM 2 + 2
= 2 · AM 2 + BC 2
2
2
1 2
c2 + b 2 = 2 · m 2 + · a
a
2
restando (*) y (**) y teniendo en cuenta que M es punto medio, o sea que
´
7.5. APLICACIONES DEL TEOREMA DE PITAGORAS

227

M B = M C,:
AB 2 − AC 2 = 4 · M B · M H
BC
· M H = 2 · BC · M H
=4
2
c2 − b 2 = 2 · a · M H

qui

A

a, D

ept

o. d

eM

atem

atic

as

Teorema 19 (Altura en funci´n de los lados).
o
En un ABC cuyos lados miden: BC = a, AC = b, AB = c; las alturas
miden:
2
ha =
p(p − a)(p − b)(p − c)
a
2
p(p − a)(p − b)(p − c)
hb =
b
2
hc =
p(p − a)(p − b)(p − c)
c
ımetro.
donde p = a+b+c =semi-per´
2

An
tio

c

ha

C

ersi

dad

a

de

H

B

b

Un
iv

Figura 24.

Demostraci´n. (Ver Figura 24.) Sea ha = AH la altura relativa al lado
o
BC, con H pueden ocurrir los siguientes casos i. B − H − C, ii. B − C − H
o H − B − C, iii. H ≡ B o H ≡ C.
Mostremos el caso i. y supongamos que c > b (ver la Figura 24.), el caso
c < b es similar, el caso c = b se deja como ejercicio; como los tri´ngulos
a
CAP´
ITULO 7. SEMEJANZA, JAIME ESCOBAR A.

228
AHB y

AHC son rect´ngulos, entonces por el teorema de Pit´goras:
a
a
c2 = h2 + BH 2
a
2
b = h2 + CH 2
a

(7.1)
(7.2)

Como B − H − C entonces HC = a − BH, sustituyendo en 7.2
(7.3)

as

b2 = h2 + (a − BH)2 = h2 + a2 + BH 2 − 2aBH
a
a

atic

y por 7.1 en la expresi´n anterior
o

atem

b2 = h2 + a2 + c2 − h2 − 2aBH = a2 + c2 − 2aBH
a
a

a2 + c 2 − b 2
2a

+

despejando h2
a
=c −

a2 + c 2 − b 2
2a

2

= c2 −

4a2 c2 − (a2 + c2 − b2 )2
(a2 + c2 − b2 )2
=
4a2
4a2

de

2

dad

h2
a

2

An
tio

c =

h2
a

qui

2

a, D

y sustituyendo en 7.1

o. d

a2 + c 2 − b 2
2a

ept

BH =

eM

como a = 0, ya que A, B, C son tres puntos distintos no colineales, despejando BH en la expresi´n anterior
o

(2ac + a2 + c2 − b2 )(2ac − a2 − c2 + b2 )
((a + c)2 − b2 )(b2 − (a − c)2 )
=
4a2
4a2
(a + c + b)(a + c − b)(b + a − c)(b − a + c)
=
4a2
(a + b + c)(a + c − b)(a + b − c)(b + c − a)
=
(7.4)
4a2

Un
iv

ersi

=

Como p =

a+b+c
2

entonces a + b + c = 2p y tambi´n
e

p−a=

a+b+c
a + b + c − 2a
b+c−a
−a=
=
2
2a
2
´
7.5. APLICACIONES DEL TEOREMA DE PITAGORAS

229

por lo tanto b + c − a = 2(p − a)
Similarmente a + b − c = 2(p − c) y a + c − b = 2(p − b), sustituyendo en 7.4
2p · 2(p − a) · 2(p − b) · 2(p − c)
4
= 2 · p · (p − a) · (p − b) · (p − c)
2
4a
a
ha =

p · (p − a) · (p − b) · (p − c).

´
CONSTRUCCIONES BASICAS

atem

7.5.1.

2
a

as

por lo tanto

atic

h2 =
a

eM

1. Dividir un segmento en n segmentos congruentes, con n entero positivo.
X

An

o. d

An−1

ept

A2

C1

C2
Cn−1
Figura 25.

B

An
tio

qui

A

a, D

A1

de

Construcci´n. Para la construcci´n, haremos los siguientes pasos cono
o
secutivos (Ver Figura 25.).

Un
iv

ersi

dad

−→
−
• Por A trazo una semirrecta AX cualesquiera, tal que A, B y X
sean tres puntos distintos no colineales.
−→
−
• Con centro en A y radio cualesquiera, trazo arco que corta a AX
en A1 .
−→
−
• Con centro en A1 y el mismo radio, trazo arco que corta a AX
en A2 de tal manera que A − A1 − A2 ; similarmente se hallan los
puntos A3 , . . . , An−1 , An .
• Uno An con B y por An−1 , An−2 , . . . , A2 , A1 trazo paralelas a An B
las cuales cortan a AB en Cn−1 , Cn−2 , . . . , C2 , C1 .
• AC1 ∼ C1 C2 ∼ · · · ∼ Cn−1 B
=
=
=
CAP´
ITULO 7. SEMEJANZA, JAIME ESCOBAR A.

230

Justificaci´n. Como
o
AA1 ∼ A1 A2 ∼ · · · ∼ An−1 An
=
=
=
y
BAn

···

Cn−1 An−1

C 1 A1

atic

AC1 ∼ C1 C2 ∼ · · · ∼ Cn−1 B
=
=
=

as

entonces por el Teorema fundamental del paralelismo (Lema 1),

eM

atem

2. Dividir un segmento dado en una proporci´n dada p , donde p, q son
o
q
enteros positivos.
X

o. d

Q

a, D

B

qui

C

Figura 26.

An
tio

A

ept

P

de

Construcci´n. Para la construcci´n, haremos los siguientes pasos cono
o
secutivos (Ver Figura 26.).

Un
iv

ersi

dad

−→
−
• Por A trazo una semirrecta AX cualesquiera, tal que A, B y X
sean tres puntos distintos no colineales.
−→
−
• Con centro en A y radio cualesquiera, trazo arco que corta a AX
en A1 , este procedimiento lo efect´o p veces hasta completar un
u
segmento AP de longitud pAA1 , a continuaci´n de este segmento
o
y utilizando la misma medida AA1 construyo el segmento P Q de
longitud qAA1 .
• Uno Q con B y por P trazo paralela a QB la cual corta a AB en
C.
• el punto C es el punto pedido.
´
7.5. APLICACIONES DEL TEOREMA DE PITAGORAS
Justificaci´n. Como QB
o
de Tales en el tri´ngulo)
a

231

P C, entonces por el Corolario 1 (Teorema
CA
pAA1
p
=
=
CB
qAA1
q

3. Hallar la cuarta proporcional de tres segmentos dados: a, b, c.

atic
B

o. d

Figura 27.

X

ept

C

eM

atem

P
A

as

Y

Q

An
tio

qui

a, D

Construcci´n. Para la construcci´n, haremos los siguientes pasos cono
o
secutivos (Ver Figura 27.).
−→
−
• Trazo una semirrecta AX cualesquiera,
−
→
• Trazo una semirrecta AY cualesquiera, que no este contenida en
←→

la recta AX

Un
iv

ersi

dad

de

−
→
• Con centro en A y radio a, trazo arco que corta a AY en P .
−
→
• Con centro en P y radio b, trazo arco que corta a AY en Q, tal
que A − P − Q.
−→
−
• Con centro en A y radio c, trazo arco que corta a AX en C.
−→
−
• Uno P con C y por Q trazo paralela a P C la cual corta a AX en
B.
• el segmento CB es el segmento pedido.
Justificaci´n. Como P C
o
de Tales en el tri´ngulo)
a

QB, entonces por el Corolario 1 (Teorema

AC
AP
=
PQ
CB

o sea

a
c
=
b
CB
232

CAP´
ITULO 7. SEMEJANZA, JAIME ESCOBAR A.

4. Dado C ∈ AB, hallar el conjugado arm´nico de C con respecto a AB.
o
P
Q

¥

¥

C

D

B

atem

atic

as

A

eM

Figura 28.

o. d

Construcci´n. Para la construcci´n, haremos los siguientes pasos cono
o
secutivos (Ver Figura 28.).

ept

• Trazo circunferencia de centro A y radio AC.

a, D

• Trazo circunferencia de centro B y radio BC.

qui

• En la circunferencia C(A, AC), trazo un radio cualesquiera AP
no paralelo a AB.

An
tio

• Por B trazo, en la circunferencia C(B, BC), el radio BQ tal que
BQ AP
←→

• Uno P con Q y prolongo hasta cortar la recta AB en D .

de

• el punto D es el conjugado de C con respecto a AB.

Un
iv

ersi

dad

Justificaci´n. Como BQ AP entonces ADP ∼ BDQ entonces,
o
teniendo en cuenta que AP y BQ son radios en las respectivas circunferencias,
DA
CA
DA
AP
=
o sea
=
BQ
DB
CB
DB
5. Dado AB y dada la proporci´n p , donde p, q son enteros positivos. Hao q
CA
DA
llar C, D conjugados arm´nicos de AB tal CB = DB = p .
o
q
Construcci´n. Para la construcci´n, haremos los siguientes pasos cono
o
secutivos (Ver Figura 29.).
´
7.5. APLICACIONES DEL TEOREMA DE PITAGORAS

233

X
P
Y
Q

¦

¦

C

A

B

D

as

R

atic

Z

atem

Figura 29.

An
tio

qui

a, D

ept

o. d

eM

←→
−→
−
• Trazo una semirrecta cualquiera AX que no este contenida en AB.
−→
−
−→ −→
−
−
• En el mismo semiplano, trazo la semirrecta BY tal que BY AX
−
→
−→
−
y trazo tambi´n la semirrecta BZ opuesta a la semirrecta BY .
e
−→
−
• Sobre la semirrecta AX y con la misma unidad de medida α, trazo
el segmento AP tal que AP = p · α.
−→
−
• Sobre la semirrecta BY y con la misma unidad de medida α, trazo
el segmento BQ tal que BQ = q · α.
−
→
• Sobre la semirrecta BZ y con la misma unidad de medida α, trazo
el segmento BR tal que BR = q · α.
←→

• Uno P con Q y prolongo hasta cortar la recta AB en D .

de

• Uno P con R el cual corta a AB en C .

AP D ∼
entonces,

y tambi´n
e

Y Z entonces

Un
iv

Justificaci´n. Como AX
o

ersi

dad

• Los puntos C y D son conjugados arm´nicos con respecto a AB
o
bajo la raz´n p .
o q

BQD

y

AP C ∼

BRC

DA
AP
=
DB
BQ

o sea

DA
p·α
p
=
=
DB
q·α
q

CA
AP
=
CB
BR

o sea

CA
p·α
p
=
=
CB
q·α
q
CAP´
ITULO 7. SEMEJANZA, JAIME ESCOBAR A.

234
luego

DA
CA
=
CB
DB

6. Hallar la media proporcional de dos segmentos a y b dados.
X

atem

atic

as

D

§

C

eM

O
B
Figura 30.

l

o. d

A

qui

• Sobre una recta l fijo un punto A.

a, D

ept

Construcci´n. Para la construcci´n, haremos los siguientes pasos cono
o
secutivos (Ver Figura 30.).

• Con centro en A y radio a trazo arco que corta a l en B.

de

An
tio

• Con centro en B y radio b trazo arco que corta a l en C, tal que
A − B − C.
−→
−
• Por B trazo BX ⊥ l.

dad

• Hallo O punto medio de AC.

ersi

• Trazo semicircunferencia de centro O y radio OA, la cual corta a
−→
−
BX en D.

Un
iv

• El segmento BD es media proporcional entre a y b.
Justificaci´n. Como AC es di´metro, entonces ACD es rect´ngulo y
o
a
a
como DB es altura relativa a la hipotenusa en dicho tri´ngulo, entonces,
a
por el Teorema 13 (Proporcionalidad en el tri´ngulo rect´ngulo)
a
a
BD2 = BA · BC = a · b
es decir BD es media proporcional entre a y b.
7.6. APLIC. DE LA SEMEJANZA A LA CIRCUNFERENCIA

7.6.

235

APLICACIONES DE LA SEMEJANZA
A LA CIRCUNFERENCIA

as

Teorema 20 (Teorema de Tolomeo).
En un cuadril´tero c´
a
ıclico, el producto de las medidas de las diagonales es
igual a la suma de los productos de las medidas de los lados opuestos.

atic

A

atem

d

c

eM

a

D
E

B

o. d

C

An
tio

qui

Figura 31.

a, D

F

ept

b

Demostraci´n. (Ver Figura 31.) Por el axioma de construcci´n de angulo,
o
o
´
−
→
existe una semirrecta AF ⊂ ←→ con F sobre la circunferencia, tal que
AB: C
DAC ∼ BAF , sea {E} = AF ∩ DB y como CAF ∼ CAF entonces por el
=
=
axioma de suma (o resta) de angulos congruentes, DAF ∼ BAC
´
=
∼ BAF y DCA ∼ ABE (por
En los ADC y AEB se tiene: DAC =
=
Teorema del angulo inscrito), entonces por el criterio A-A:
´

Un
iv

ersi

dad

de

π

ADC ∼

luego

ADC
:
AEB

AEB

AD
AC
DC
=
=
AE
AB
EB

luego
AC · EB = DC · AB

(∗)
236

CAP´
ITULO 7. SEMEJANZA, JAIME ESCOBAR A.

En los DAE y ABC se tiene: DAE ∼ BAC y ADE ∼ ACB (por
=
=
Teorema del angulo inscrito), entonces por el criterio A-A:
´
DAE ∼

ABC

luego
DA
DE
AE
=
=
AC
BC
AB

DAE
:
ABC

(∗∗)

atic

DA · BC = DE · AC

as

luego

atem

sumando (*) y (**):

eM

DC · AB + DA · BC = AC · EB + DE · AC = AC(EB + DE) = AC · BD
AC · BD = a · c + b · d.

o. d

es decir,

An
tio

qui

a, D

ept

Teorema 21.
Si dos cuerdas se interceptan en el interior de una circunferencia entonces
el producto de las medidas de los segmentos determinados por el punto de
intersecci´n en una de las cuerdas es igual al producto de las medidas de
o
los segmentos determinados en la otra cuerda.
C

O

ersi
Un
iv

D

¨

X

dad

de

A

B

Figura 32.

Demostraci´n. (Ver Figura 32.) Sean AB y CD cuerdas tales que {X} =
o
AB ∩ CD y A − X − B y C − X − D. En los AXC y BXD se tiene
7.6. APLIC. DE LA SEMEJANZA A LA CIRCUNFERENCIA

237

que: por opuestos por el v´rtice AXC ∼ BXD y por el Teorema del angulo
e
´
=
∼ XDB, luego por el criterio A-A,
inscrito CAX =
AXC ∼
=

BXD

luego
XA
AC
XC
=
=
XD
BD
XB

as

AXC
:
BXD

o. d

eM

atem

atic

o sea que XA · XB = XC · XD
Nota: 1.) Obs´rvese que si por ejemplo el punto X ≡ A ≡ C, es decir, los
e
dos segmentos se cortan sobre la circunferencia, entonces tambi´n se cumple
e
que XA · XB = XC · XD = 0.
2.) El resultado de este teorema nos muestra que para cualquier cuerda que
pase por el punto X se cumple que XA · XB permanece constante o sea que
este producto no depende de la cuerda, sino del punto X.

a, D

ept

El siguiente teorema se deja como ejercicio, es el rec´
ıproco del teorema
anterior.

de

An
tio

qui

Teorema 22.
Si dos segmentos se interceptan en un punto que esta en el interior de los dos
segmentos y el producto de las medidas de los segmentos determinados por
el punto de intersecci´n en el primer segmento es igual al producto de las
o
medidas de los segmentos determinados por el punto en el segundo segmento, entonces los extremos de los segmentos est´n sobre una circunferencia.
a

Un
iv

ersi

dad

Teorema 23.
Si desde un punto X exterior a una circunferencia se trazan dos semirrectas
secantes l y m que cortan a la circunferencia en A, B y C, D respectivamente,
entonces
XA · XB = XC · XD
Demostraci´n. (Ver Figura 33.) Por el Teorema del angulo inscrito BAD ∼
o
´
=
BCD y el X es com´n para los XAD y XBC entonces por el criterio
u
A-A
XAD ∼ XBC
CAP´
ITULO 7. SEMEJANZA, JAIME ESCOBAR A.

238

A

B
©

O

C
D

as

X

luego

eM

XA
XD
AD
=
=
XC
XB
BC

luego

ept

XA · XB = XC · XD

o. d

XAD
:
XBC

atem

atic

Figura 33.

qui

a, D

Nota: El resultado de este teorema nos muestra que para cualquier semirrecta que pase por el punto X se cumple que XA · XB permanece constante
o sea que este producto no depende de la semirrecta, sino del punto X.

An
tio

El rec´
ıproco del anterior teorema tambi´n es cierto, se deja como ejercicio.
e
Teorema 24 (Rec´
ıproco).

dad

de

Si desde un punto X se trazan dos semirrectas l y m y A, B son puntos de
l y C, D son puntos de m, tales que

ersi

XA · XB = XC · XD,

Un
iv

entonces los puntos A, B, C, D est´n sobre una circunferencia.
a
Teorema 25.
Si desde un punto exterior a una circunferencia se trazan dos semirrectas,
una tangente y la otra secante, entonces el segmento entre el punto y el
punto de tangencia es media proporcional entre los segmentos determinados
entre el punto exterior y los puntos de intersecci´n de la secante con la
o
circunferencia.
7.7. EJE RADICAL Y SUS PROPIEDADES

239

A

B


O

M
X

as

C

atem

atic

Figura 34.

Demostraci´n. (Ver Figura 34.) Como por el teorema del angulo semio
´

XAC ∼

eM

ept

XBC,

a, D

XAC
:
XBC

XBC, en-

XA
XC
AC
=
=
XC
XB
BC

qui

luego

XAC y

o. d

1
inscrito el BCX = 2 CM B y X es com´n para los
u
tonces por el criterio A-A,

luego

EJE RADICAL Y SUS PROPIEDADES

de

7.7.

An
tio

XA · XB = XC · XC = XC 2 .

Un
iv

ersi

dad

Definici´n 9 (Potencia de un punto con respecto a una circunfeo
rencia). La potencia de un punto X con respecto a una circunferencia
C(O, r) es el producto XA · XB, donde A y B son los puntos de intersecci´n
o
de la circunferencia con una recta que pasa por X.
Notaci´n: la potencia del punto X con respecto a la circunferencia
o
C(O, r) se denota por pX;O , es decir,
pX;O = XA · XB
Nota a.) De acuerdo a los teoremas 21 y 23, todas las rectas que pasan por el
punto X tienen igual potencia, por lo tanto, la potencia depende solamente
240

CAP´
ITULO 7. SEMEJANZA, JAIME ESCOBAR A.

del punto y la circunferencia.
b.) Si X es un punto exterior a la C(O, r) y d es la distancia del punto X al
centro O de la circunferencia, entonces (ver la Figura 35.)
pX;O = XA · XB = (XO + OA)(XO − OB) = (d + r)(d − r) = d2 − r2
←→

O






B

X

a, D

qui

Figura 35.

ept

o. d

eM



A

atem

atic

as

donde A, B son los puntos de intersecci´n de la recta XO con la C(O, r).
o
En este caso pX;O  0, ya que d  r

de

An
tio

c.) Con el punto X y la circunferencia C(O, r) pueden suceder tres casos:
1. X ∈ ExtC(O, r), en este caso vimos que pX;O  0, ya que d  r
2. X ∈ IntC(O, r), en este caso pX;O = d2 − r2  0, ya que d  r
3. X ∈ C(O, r), en este caso pX;O = d2 − r2 = 0, ya que d = r

ersi

dad

En resumen, la potencia es positiva en el exterior de la circunferencia,
negativa en el interior de la circunferencia y es cero cuando el punto esta
sobre la circunferencia.

Un
iv

d.) Si X ≡ O, entonces d = 0 y por tanto pX;O = d2 − r2 = −r2 , este es el
valor m´
ınimo de la potencia, ya que d = 0 es el valor m´
ınimo de d.
e.) Por el teorema 25, la potencia de un punto exterior a una circunferencia
es igual al cuadrado de la medida del segmento tangente desde el punto X
a la circunferencia C(O, r), es decir, pX;O = XT 2 , donde T es el punto de
tangencia.
7.7. EJE RADICAL Y SUS PROPIEDADES

241

f.) La potencia de un punto interior a una circunferencia es igual y negativa,
del cuadrado de la semi-cuerda perpendicular al di´metro que pasa por el
a
punto.
C

as

O


B

atic

X

atem

A

eM

D

ept

o. d

Figura 36.

An
tio

qui

a, D

(Ver Figura 36.) En efecto, sea AB di´metro y X ∈ AB y sea CD una
a
cuerda tal que CD ∩ AB = {X} y AB⊥CD, por tanto X es punto medio de
CD, entonces
CD
2

2

de

pX;O = −XA · XB = −XC · XD = −XC 2 = −XD2 = −

Un
iv

ersi

dad

Teorema 26 (Teorema del eje radical).
El lugar geom´trico de los puntos de igual potencia con respecto a dos
e
circunferencias no conc´ntricas, es una recta perpendicular a la recta que
e
pasa por los centros.

Demostraci´n. (Ver Figura 37.) Sean las circunferencias C(O, r) y C(O , r ),
o
sea M el punto medio de OO y sea X un punto tal que pX;O = pX;O (∗), sea
←→

H la proyecci´n de X sobre OO , veamos que cualquiera que sea el punto
o
←→

X con la propiedad (*), tendr´ como proyecci´n sobre OO el punto H.
a
o
En efecto, por la hip´tesis, por la propiedad b.) hecha en la nota anterior
o
CAP´
ITULO 7. SEMEJANZA, JAIME ESCOBAR A.

242

X

H






M

O’

atic

as

O

eM

atem

Figura 37.

o. d

y por el Teorema 18 b), se tiene
pX;O = pX;O

(XO) − (r)2 = (XO )2 − (r )2 por la propiedad b)
(XO)2 − (XO )2 = (r)2 − (r )2
2 · OO · M H = (r)2 − (r )2 por el Teorema 18 b)
(r)2 − (r )2
luego M H =
2 · OO

An
tio

qui

a, D

ept

2

de

como r, r , OO son constantes y OO = 0, entonces M H es constante y como
M es fijo entonces H es fijo, cualquiera sea el punto X, por lo tanto los puntos
X que cumplen con la propiedad (*) est´n sobre una recta perpendicular a
a

dad

←→

OO .

Un
iv

ersi

La recta cuya existencia esta garantizada por el anterior teorema, le
damos el siguiente nombre:
Definici´n 10 (Eje Radical). La recta cuyos puntos tienen igual potencia
o
con respecto a dos circunferencias, se le llama Eje Radical.
Propiedades del Eje Radical.
1. Si las dos circunferencias se interceptan, entonces el eje radical pasa
por los puntos de intersecci´n, ya que cada punto de intersecci´n tiene
o
o
potencia igual a cero con respecto a las dos circunferencias.
7.7. EJE RADICAL Y SUS PROPIEDADES

243

2. Si las dos circunferencias son tangentes, entonces el eje radical es la
tangente com´n a ambas circunferencias, ya que la potencia en el punto
u
de tangencia es cero con respecto a las dos circunferencias y la tangente
com´n es perpendicular a la recta que pasa por los centros de las dos
u
circunferencias.

as

3. Si las dos circunferencias son conc´ntricas y distintas, entonces no hay
e
2
2
2
eje radical, ya que d − r = (d ) − (r )2

atic

Teorema 27 (Propiedades del Eje Radical).

atem

a.) Las tangentes desde un punto del Eje Radical a las dos circunferencias,
son congruentes.

ept

o. d

eM

b.) Los Ejes Radicales de tres circunferencias, cuyos centros son no colineales, tomados de dos en dos, son concurrentes, (este punto de concurrencia se le llama Centro Radical).

An
tio

qui

a, D

Demostraci´n. a.) (ver Figura 38.) Sea X un punto del Eje Radical y sean
o
XT y XT1 tangentes a las circunferencias C(O, r) y C(O , r ) en T y T1
2
respectivamente, entonces por el Teorema 25 pX;O = XT 2 y pX;O = XT1 y
como X pertenece al Eje Radical, entonces pX;O = pX;O luego
2
XT 2 = XT1

de

luego XT = XT1 , o sea que XT ∼ XT1
=
b.)(ver Figura 39.) Sea l el Eje Radical de C(O, r) y C(O , r ) y sea l el
←→

←→

ersi

dad

Eje Radical de C(O , r ) y C(O , r ) por lo tanto l ⊥OO y l ⊥O O .
Como O, O , O son no colineales, entonces l y l se interceptan, sea

Un
iv

{X} = l ∩ l .

Veamos que X ∈ l .
En efecto, como X ∈ l entonces

pX;O = pX;O
y como X ∈ l entonces

pX;O = pX;O

(∗)
(∗∗)
244

CAP´
ITULO 7. SEMEJANZA, JAIME ESCOBAR A.
X
T


T1


H




O

atic

as

O’

eM

atem

Figura 38.

o. d

l

O’



ept



l

An
tio

qui

X

a, D

O

l



de

O”

ersi

entonces de (*) y (**)

dad

Figura 39.

luego X ∈ l .

Un
iv

pX;O = pX;O

Observaci´n.
o
De la parte b.) del teorema anterior se concluye que:
1. Si las tres circunferencias son secantes dos a dos, entonces las cuerdas comunes son concurrentes.
2. Si las tres circunferencias son tangentes dos a dos, entonces las tangentes
7.7. EJE RADICAL Y SUS PROPIEDADES

245

comunes son concurrentes.
Con el Eje Radical se pueden hacer construcciones de circunferencias.
Ejemplo. Construir una circunferencia que pase por dos puntos y sea tangente a una recta dada.
Demos el problema por construido. Supongamos que los puntos dados son
←→

as

A, B y la recta dada es l, se presentan dos situaciones: a) AB ∩l = ∅,
←→

atic

←→

b) AB ∩l = ∅.

←→

atem

a) Si AB ∩l = ∅, sea {X} =AB ∩l = ∅, sea C(O, r) la circunferencia buscada y sea C(O , r ) una circunferencia cualesquiera que pase por A y B,
←→

a, D

ept

o. d

eM

entonces AB es el Eje Radical de estas dos circunferencias, por lo tanto las
tangentes desde el punto X a las dos circunferencias son congruentes; si XT
es la tangente a la C(O , r ) y XT es la tangente a la circunferencia buscada
C(O, r), entonces XT = XT .

O’

An
tio

B




T1



Un
iv

X


T

ersi

dad

de



O

T’


A

qui

m

Figura 40.

Construcci´n. Para la construcci´n, haremos los siguientes pasos consecuo
o
tivos (Ver Figura 40.).
Uno A con B y prolongo hasta cortar l en X.
CAP´
ITULO 7. SEMEJANZA, JAIME ESCOBAR A.

246

Trazo m la mediatriz de AB.
Por un punto cualesquiera O de m, trazo una circunferencia que pase
por A, B.
Desde X trazo XT tangente a la circunferencia de centro O
Con centro en X y radio XT trazo arcos que cortan a l en T y T1 .

←→

atic

as

Las circunferencias que pasan por A, B, T y por A, B, T1 son las circunferencias pedidas (dos soluciones).
←→

←→

atem

b) Si AB ∩l = ∅, luego AB l. Sea C(O, r) la circunferencia buscada y sea
T el punto de tangencia entre la C(O, r) y l, por lo tanto OT ⊥ l, pero como
←→

←→

eM

AB l, entonces OT ⊥ AB, luego OT es mediatriz de AB

ept

o. d

Construcci´n. Para la construcci´n, haremos los siguientes pasos consecuo
o
tivos.

a, D

Uno A con B .

Trazo m la mediatriz de AB que corta a l en T .

An
tio

qui

Trazo circunferencia que pasa por A, B, T , que es la circunferencia pedida.

ersi

dad

de

Ejemplo. Construir una circunferencia que pase por dos puntos y sea tangente exteriormente a una circunferencia dada.
Demos el problema por construido. Supongamos que los puntos dados son
A, B y la circunferencia dada es C(O , r ) y sea m la mediatriz de AB, se
presentan dos casos: a) O ∈ m, b) O ∈ m
/

Un
iv

a) O ∈ m, sea C(O , r ) una circunferencia que pase por A, B e inter/
←→

cepte a la circunferencia dada C(O , r ) en los puntos C, D, por lo tanto CD
es el Eje Radical de estas dos circunferencias, como la circunferencia buscada C(O, r) y la circunferencia dada C(O , r ) son tangentes, entonces la
tangente l com´n a estas dos circunferencias es el Eje Radical de ambas y
u
←→

como O ∈ m, entonces l y CD se interceptan en un punto X; como los Ejes
/
Radicales de tres circunferencias cuyos centros no son colineales son concurrentes, entonces X es el centro radical de las tres circunferencias, luego las
7.7. EJE RADICAL Y SUS PROPIEDADES

247

m


A

O




atem



D


C

atic

X

T

l

as



O”

B

O’

eM



ept

o. d

T1
Figura 41.

a, D

tangentes desde X a las tres circunferencias son congruentes.

qui

Construcci´n. Para la construcci´n, haremos los siguientes pasos consecuo
o
tivos (Ver Figura 41.).

An
tio

Uno A con B .
Trazo m la mediatriz de AB .

dad

de

Por un punto O de m trazo circunferencia que pasa por A, B y que
corte a la circunferencia dada C(O , r ) en los puntos C, D.
←→

ersi

Uno C con D y prolongo hasta cortar AB en X.

Un
iv

Desde X trazo trazo XT y XT1 tangentes a la circunferencia dada
C(O , r ).
Las circunferencias que pasan por A, B, T y A, B, T1 son las circunferencias pedidas (dos soluciones).
b) Si O ∈ m. Sea {T } = m ∩ C(O , r ), en este caso, O, T, O son colineales
y por tanto T es el punto de tangencia.
248

CAP´
ITULO 7. SEMEJANZA, JAIME ESCOBAR A.

Construcci´n. Para la construcci´n, haremos los siguientes pasos consecuo
o
tivos.
Uno A con B .
Trazo m la mediatriz de AB, la cual intercepta a la circunferencia dada
C(O , r ) en T .

Un
iv

ersi

dad

de

An
tio

qui

a, D

ept

o. d

eM

atem

atic

as

Trazo circunferencia que pase por los puntos A, B, T y esta es la circunferencia pedida.
7.8. EJERCICIOS Y PROBLEMAS DE SEMEJANZA

7.8.

249

Ejercicios y Problemas de Semejanza

1. Sea ∆ABC un tri´ngulo inscrito en la circunferencia C(O, r), sea AD,
a
con D ∈ C(O, r), la bisectriz del BAC y sea {E} = BC ∩ AD. Mostrar
que a) BD 2 = AD · ED, b) ∆BED ∼ ∆AEC
B

atem

atic

as

2. En la figura, si ABD ∼
=
DBE ∼ EBC, entonces
=
AD
= AB·BD .
EC
BE·BC
A

C

E

B

N

K

a, D

M
D

C

qui

H

3. Si ABCD es un paralelogramo y M N AB, AB =
12, DM = 4, DE = 6,
KB = 2KH. Hallar: a)
AM , b) DH, c) DC, d)
KF , d) LM , e) M N .

ept

A

o. d

eM

D

An
tio

E

dad

de

4. Sea ABC un tri´ngulo cualesquiera, por el v´rtice A trazamos una
a
e
−→
−
semirrecta AX paralela al lado BC. Desde M punto medio de BC
−→
−
se traza una recta cualesquiera que corta a AX en N , AC en P y la
prolongaci´n de AB en Q. Probar que
o

Un
iv

ersi

QN
PN
=
PM
QM

5. Demostrar que el cuadrado de la medida de la bisectriz AE de un
angulo exterior de un ∆ABC es igual al producto de las medidas de
´
los segmentos que la bisectriz determina sobre la recta que contiene al
lado opuesto, menos el producto de las medidas de los otros dos lados.
(Ayuda: siendo C(O, r) la circunferencia que circunscribe al tri´ngulo
a
←→

y {D} = C(O, r)∩ AE, observar los ∆DAC y ∆ABE).
250

CAP´
ITULO 7. SEMEJANZA, JAIME ESCOBAR A.

6. Se tiene un cuadrado ABCD de lado a. Se traza una circunferencia que
pasa por el v´rtice A y por los puntos medios de los lados AB y AD.
e
Probar que la medida de una tangente a dicha circunferencia trazada
desde el punto C es igual a a.
7. Construir un tri´ngulo dadas las raz´n entre los lados c y b (es decir,
a
o
p
c
dado b = q ), la mediana ma y el lado a ( c = p , ma , a)
b
q

atem

atic

as

8. Por un punto D del lado AB de un ABC se traza DE AC (E sobre
BC), de tal manera que DB = e, CE = 2e, BE = 2AD. Calcular los
lados AB y BC del tri´ngulo.
a

eM

9. Demostrar que en un mismo tri´ngulo las alturas son inversamente
a
proporcionales a sus respectivos lados.

ept

o. d

a
10. Considere la C(O, r). Sea AB un di´metro. Se traza por B una tangente
y por A una secante cualesquiera que corta a la C(O, r) en M y a la
tangente en N . Probar que AM.AN = 4r 2 .

qui

a, D

11. Sea la C(O, r) y AB di´metro y sea M un punto en la prolongaci´n
a
o
de AB, se trazan las tangentes M N y M P a la C(O, r), la cuerda N P
corta al di´metro AB en C. Demostrar que
a

An
tio

MA
CA
=
CB
MB

Un
iv

ersi

dad

de

12. Sea C(O, r), se traza una cuerda CD, O el punto medio de CD, se
traza la circunferencia de centro O y di´metro CD, sea AB di´metro
a
a
de C(O, r) perpendicular a CD; se trazan AT y AT tangentes a la
C(O ), la cuerda T T corta a AB en F . Demostrar que O es punto
medio de BF .
13. En ABC rect´ngulo en A la hipotenusa mide a y la altura relativa
a
a la hipotenusa mide h, se inscribe un cuadrado con un lado sobre la
hipotenusa. Calcular el lado del cuadrado en t´rminos de a y h.
e
14. En una circunferencia de di´metro 40cm. , hallar la medida de la mayor
a
y la menor cuerda que puede trazarse por un punto situado a 12cm.
del centro. Explicar porque es la mayor y la menor.
7.8. EJERCICIOS Y PROBLEMAS DE SEMEJANZA

251

15. Desde el punto medio D del lado AB del ABC, rect´ngulo en A, se
a
o
traza DE ⊥ BC, con E ∈ BC. Demostrar la relaci´n
EC 2 − EB 2 = AC 2

←→

atic

CD, demuestre que

atem

cia que circunscribe al tri´ngulo y F ∈ C(O, r)
a
ADC ∼ F BC).

as

16. Demostrar que el cuadrado de la bisectriz de un angulo exterior de un
´
tri´ngulo es igual al producto de los segmentos que la bisectriz determia
na en el lado opuesto menos el producto de los otros dos lados (Ayuda:
si CD es la bisectriz exterior en el ABC y C(O, r) es la circunferen-

eM

17. En un ABC is´sceles con AB = AC, se traza CD ⊥ AB. Demostrar
o
la relaci´n
o

o. d

AB 2 + BC 2 + CA2 = BD2 + 2DA2 + 3CD2

An
tio

qui

a, D

ept

18. Si el tri´ngulo del ejercicio anterior fuera un tri´ngulo equil´tero, mostrar
a
a
a
que las suma de los cuadrados de las medidas de los lados es igual a
cuatro veces el cuadrado de la medida de la altura.
−
−
→
19. El ABC esta inscrito en una C(O, r), sea AD la bisectriz de A con
−
−
→
D ∈ C(O, r) y sea E ∈ BC ∩ AD. Mostrar que:
a) BD2 = AD.ED, b) BED ∼ AEC.
←→

←→

20. LM N T es un paralelogramo, LT = 15, LM = 8, RN = 12, N R⊥LR,
←→

←→

de

T H⊥M N , H ∈ M N . Hallar T H.
←→

←→

←→

ABC, sea AN BC y M punto medio de BC, sea P ∈N M
←→

←→

dad

21. Dado el

ersi

∩ AB y Q ∈N M ∩ AC . Demostrar que

Un
iv

PN
QN
=
PM
QM

22. Dado un ABC is´sceles con CA ∼ CB y la circunferencia tangente
o
=
a los lados congruentes en A y B. Desde un punto M del arco de la
circunferencia en el interior del tri´ngulo, se traza M D ⊥ AB, M F ⊥
a
CB y M E ⊥ CA. Mostrar que
M D2 = M E.M F
252

CAP´
ITULO 7. SEMEJANZA, JAIME ESCOBAR A.

23. Sean AA , BB , CC las alturas de un
en el punto H. Demostrar que:

ABC; estas alturas se cortan

AA .A H = A C.A B, BB .B H = B A.B C, CC .C H = C B.C A
24. Se d´ una circunferencia de centro O y di´metro AB, por un punto M
a
a
sobre la prolongaci´n de AB, se trazan las tangentes M N y M P a la
o
circunferencia, la cuerda N P corta al di´metro en C. Demostrar que:
a

atic

as

CA
MA
=
CB
MB

eM

atem

25. Demostrar que si dos tri´ngulos tienen sus lados respectivamente paraa
lelos o respectivamente perpendiculares, entonces dichos tri´ngulos son
a
semejantes.

o. d

26. Dado un paralelogramo ABCD, tal que: DC = 32, AD = 17, AC = 28.
Hallar DB.
AB =

a, D

ept

27. Sea ∆ABC con CE, BD, AF bisectrices. Si CA = 32,
20, CB = 36. Hallar AE, CF, AD.

An
tio

qui

28. Demostrar que la suma de las longitudes de los catetos de un tri´ngulo
a
rect´ngulo, no excede la longitud de la diagonal de un cuadrado consa
truido sobre la hipotenusa del tri´ngulo como lado.
a

de

29. Demostrar que en un paralelogramo la suma de los cuadrados de los
lados es igual a la suma de los cuadrados de las diagonales.

ersi

dad

30. Sea un tri´ngulo rect´ngulo ABC (recto en A), donde: AB = 8, AC =
a
a
15 . Calcular BC, la altura AH y los segmentos BH y HC. Se traza
por B una paralela a AC que corta la altura AH en I. Evaluar AH, HI
y BI.

Un
iv

31. Sobre el lado AB de un angulo BAC, se toman dos puntos D y E y
´
por esos puntos se trazan dos paralelas que cortan al lado AC en F y
G respectivamente; se trazan F E y por el punto G, una paralela a F E
que corta a AB en H. Demostrar que AE 2 = AD.AH.
32. Dado un cuadril´tero ABCD, sea O el punto de intersecci´n de sus
a
o
diagonales. Por el punto O se traza una paralela a BC que corta a AB
en E; luego se traza por O una paralela a CD que corta a AD en F .
7.8. EJERCICIOS Y PROBLEMAS DE SEMEJANZA
AE
a. Mostrar que AB =
una misma raz´n).
o

b. Mostrar que EF

AF
AD

253

(comparar cada una de estas razones con

BD.

c. Se traza OG AB y cortando BC en G y OH
en H. Mostrar que CG.DH = BG.CH.

AD, corta a DC

atic

as

33. Demostrar que las paralelas a los lados de un tri´ngulo ABC, trazadas
a
por el punto G de concurrencia de las medianas, dividen cada lado en
tres partes iguales.
←→

←→

atem

34. Sea ABCD un cuadril´tero, sea F sobre AC y E sobre DB tales que
a
F B||DC y EC||AB. Mostrar que AD||F E.

o. d

eM

35. El per´
ımetro de un tri´ngulo mide 90 cm.. Sabiendo que las medidas
a
de los lados est´n en la relaci´n 1 : 2 : 3. Calcular la medida de cada
a
o
lado.

a, D

ept

36. Demuestre que en tri´ngulos semejantes las alturas hom´logas, las mea
o
dianas hom´logas y las bisectrices hom´logas son proporcionales a los
o
o
lados hom´logos.
o
A

dad



de

An
tio

qui

37. En la figura, la C(O, x) esta inscrita en el
sector circular ABC. Si m(ABC) = 60o , hallar x en funci´n de r.
o
r
(Rta.: 3 ) .

x

O

B

C
r

Un
iv

ersi

38. Si en un tri´ngulo rect´ngulo, X y Y son las medidas de los catetos y
a
a
Z es la medida de la altura correspondiente a la hipotenusa, demuestre
que:
1
1
1
+ 2 = 2
X2 Y
Z
a
a
39. Los catetos AB y AC de un tri´ngulo rect´ngulo ∆ABC miden respectivamente 4a y 3a. Por el punto medio M de AB se traza hacia el
exterior del tri´ngulo, un segmento M N perpendicular a AB e igual a
a
su mitad. Hallar la medida de N C.
CAP´
ITULO 7. SEMEJANZA, JAIME ESCOBAR A.

254

40. Los lados de un tri´ngulo miden 10, 12 y 18. Si el per´
a
ımetro de un
tri´ngulo semejante a ´l mide 1,200, cuales son las medidas de los lados
a
e
del segundo tri´ngulo? Cu´nto miden las tres alturas, las tres medianas
a
a
y las tres bisectrices del √
primer tri´ngulo?
√a
√
(Rta.: 300, 360, 540, 30 41, 30 176, 30 209)
A
B

F

E

C

m(BEC) = m(BF C) + m(BDC)

atem

D

atic

as

41. Si ABCD es un rect´ngulo de laa
dos a y 3a. Demostrar que

eM

42. a1 , b1 , c1 son puntos medios de los lados del tri´ngulo ∆ABC. Dea
muestre: ∆ABC ∼ ∆a1 b1 c1 ∼ ∆Ac1 b1 ∼ ∆Bc1 a1 ∼ ∆Cb1 a1

ept

o. d

43. ABCD es un paralelogramo O ∈ AC, OX ⊥ AD, OY ⊥ AB. DeAB
mostrar que OX = AD
OY
44. Dos circunferencias son tangentes interiormente en el punto A. Del
←→

←→

qui

a, D

punto A, se trazan las secantes AC y AE. B y D pertenecen a la
circunferencia interior. C y E pertenecen a la circunferencia exterior.
Demuestre que ∆ABD ∼ ∆ACE.

An
tio

45. Sea AB un di´metro en la C(O, r), por B se traza una tangente a la
a
circunferencia y por A se traza una secante cualquiera que intercepta
la circunferencia en M y a la tangente en N . Demostrar que

dad

de

AM · AN = 4r 2

Un
iv

ersi

46. Demostrar que en un trapecio el segmento paralelo a las bases que
pasa por el punto de intersecci´n de las diagonales, es bisecado por
o
dicho punto.
47. Dos tri´ngulos rect´ngulos son semejantes. Si los catetos hom´logos
a
a
o
miden a y a , b y b y las hipotenusas hom´logas miden c y c , demostrar
o
que aa + bb = cc .
48. Sean AB y CD dos cuerdas perpendiculares de una circunferencia de
radio r y sea {X} = AB ∩ CD. Demostar que
XA2 + XB 2 + XC 2 + XD2 = 4r2
7.8. EJERCICIOS Y PROBLEMAS DE SEMEJANZA

255

49. Las bases mayor y menor de un trapecio miden a y b respectivamente.
Por un punto de uno de los lados no paralelos se traza un segmento
paralelo a las bases. El segmento divide al lado en la relaci´n m : n.
o
Calcular la longitud del segmento.
50. Dado el
←→

←→

ABC, se consideran los puntos D, E, F sobre las rectas

←→

←→

←→

←→

atem

atic

as

BC, AC, AB respectivamente. Si las rectas AD, BE y CF pasan por
el centro O de la circunferencia circunscrita del ABC, cuyo radio es
R, mostrar que
1
1
2
1
+
+
=
AD BE CF
R

eM

51. En un tri´ngulo el punto de concurrencia de: las alturas, el de las
a
medianas y el de las mediatrices est´n alineados (Recta de Euler ).
a

o. d

52. Demostrar que en todo tri´ngulo, la bisectriz se encuentra entre la
a
mediana y la altura trazadas desde el mismo v´rtice.
e

a, D

ept

53. Las bases de un trapecio miden 20 y 12 y los lados no paralelos miden
10 y 12. Calcular la medida de las diagonales y de las alturas y los lados
del tri´ngulo que se forma al prolongar los lados no paralelos.
a

An
tio

qui

54. ABCD es un cuadril´tero. AB = a, BC = b, CD = c, DA = d, CE =
a
EA = m, BF = F D = n, EF = r.
Demuestre: a2 + b2 + c2 + d2 = (2m)2 + (2n)2 + 4r2 .

dad

de

55. Dados dos segmentos de longitud a cm. y b cm., construir con regla y
comp´s:
a
a) un segmento de longitud ab cm.
b) un segmento de longitud a cm.
b

ersi

56. Trazar las tangentes exteriores y las interiores a dos circunferencias.

Un
iv

57. Constru´ un tri´ngulo ABC, conociendo
ır
a
a) BC, ABC y BN que es la altura desde B, (a, β, hb ).
b) BC, AM y AH que son la mediana y la altura correspondientes a
BC, (a, ma , ha ).
c) BC, y la altura y la bisectriz BH y CD, (a, hb , vc ).
d ) BC y las alturas BH y CP , (a, hb , hc ).
CAP´
ITULO 7. SEMEJANZA, JAIME ESCOBAR A.

256

e) BC, AC y la altura BH, (a, b, hb ).
f ) BC, BAC y la mediana AM , (a, α, ma ).
g) BC, BAC y la altura BH, (a, α, hb ).
h) Los pies de las tres medianas.
i ) Las tres medianas: ma , mb , mc .

as

j ) ABC, ACB y el per´
ımetro , (β, γ, p; donde p = a + b + c).

atem

atic

58. Construir un tri´ngulo equil´tero, conociendo el radio de la circunfea
a
rencia inscrita.

eM

59. Construir un tri´ngulo equil´tero, conociendo su per´
a
a
ımetro.

o. d

60. Construir un tri´ngulo is´sceles conociendo el per´
a
o
ımetro y la medida
de la altura correspondiente a la base.

a) AB

a, D

ept

61. Construir una circunferencia que pase por dos puntos A y B y que sea
tangente a una recta l; con A y B del mismo lado con respecto a l.
l,

qui

b) AB ∩ l = {P }.

An
tio

62. Construir una circunferencia que sea tangente a dos rectas paralelas
dadas y que pase por un punto dado.

dad

de

63. Construir una circunferencia que sea tangente a dos rectas que se cortan
y pase por un punto en el interior del angulo entre las dos rectas.
´

ersi

64. Construir una circunferencia que sea tangente a una circunferencia y a
una recta dadas y que pase por un punto dado.

Un
iv

65. Dado un punto en el interior de una circunferencia, construir una cuerda
tal que el punto dado sea punto medio de dicha cuerda.
66. Sea AB di´metro de una circunferencia, A, B, M colineales con B entre
a
A y M , M N tangente en N y N C ⊥ AB, C entre A y B. Mostrar que
MA
CA
=
CB
MB
7.8. EJERCICIOS Y PROBLEMAS DE SEMEJANZA

257

67. Dado un angulo XOY y un punto A en el interior de XOY , trazar por
´
−→
−
−
−
→
A una recta que corte a OX en M y a OY en N , de tal forma que A
sea punto medio de M N .
68. Dos circunferencias de centros O y O1 y de radios diferentes son secantes
en A. Trazar por A una cuerda BC, de tal forma que A sea el punto
medio de BC. (B ∈ C(O) y C ∈ C(O1 ) ).

atic

as

69. Constru´ un tri´ngulo conociendo dos angulos y la suma de las medidas
ır
a
´
de dos de sus lados.

atem

70. Construir un rect´ngulo ABCD conociendo AB y el angulo AOB fora
´
mado por las diagonales.

o. d

eM

71. Construir un tri´ngulo ABC, rect´ngulo en A, conociendo la suma de
a
a
las medidas de los catetos y el angulo C.
´

ept

72. Construir un rect´ngulo conociendo su per´
a
ımetro y su diagonal.

a, D

73. Construir un trapecio conociendo sus bases y sus diagonales.
74. Construir un cuadril´tero conociendo sus lados y una de sus diagonales.
a

An
tio

qui

75. Construir un cuadril´tero inscriptible conociendo BD, y AC que son
a
sus diagonales, el angulo A y el lado AB.
´

de

76. Circunscribir un tri´ngulo equil´tero en una circunferencia de radio
a
a
dado.

dad

77. Construir una circunferencia que sea tangente a dos rectas dadas y cuyo
centro est´ sobre una recta dada.
e

ersi

78. Construir una circunferencia tangente a tres rectas dadas.

Un
iv

79. Trazar una recta tangente a una circunferencia dada y paralela a una
recta dada.
80. Construir un tri´ngulo conociendo:
a
a) Los pies E, F, D de las tres alturas.
b) Un lado BC, el angulo opuesto α , y la suma o la diferencia de
´
los otros dos lados (a, α, c − b), (a, α, c + b).
258

CAP´
ITULO 7. SEMEJANZA, JAIME ESCOBAR A.
c) Un angulo β y las alturas opuestas AD y CF . (β, ha , hc ).
´
d ) Un angulo β , la altura BE y la altura AD, (β, hb , ha ).
´
´
e) Un lado BC, un angulo β , y la mediana AD (a, β, ma ).
f ) El per´
ımetro, un angulo y la altura bajada desde el v´rtice del
´
e
angulo: (p, α, ha ).
´

as

g) La altura y bisectriz bajadas del mismo v´rtice y el radio de la
e
circunferencia inscrita (vc , hc , r).

atem

atic

h) La altura y la mediana bajadas desde el mismo v´rtice y el radio
e
de la circunferencia circunscrita (ma , ha , R).
81. Construir un tri´ngulo conociendo:
a

o. d

eM

a) Dos lados y la longitud de la bisectriz del angulo comprendido
´
(a, c, vb ).

ept

b) La base AB , el angulo opuesto y la suma de las medidas de los
´
lados que comprenden este angulo (c, γ, a + b).
´

Un
iv

ersi

dad

de

An
tio

qui

a, D

82. Por un punto P exterior a una circunferencia trazar una secante P AB,
PA
tal que AB = m donde m, n son dos n´meros naturales dados.
u
n

Más contenido relacionado

La actualidad más candente

La actualidad más candente (18)

Teorema de tales
Teorema de talesTeorema de tales
Teorema de tales
 
Geom t roporcionalidad (1)
Geom t roporcionalidad (1)Geom t roporcionalidad (1)
Geom t roporcionalidad (1)
 
2 desarrollo de_clase_2
2 desarrollo de_clase_22 desarrollo de_clase_2
2 desarrollo de_clase_2
 
Capítulo 4 (pp. 294 339) word
Capítulo 4  (pp. 294 339) wordCapítulo 4  (pp. 294 339) word
Capítulo 4 (pp. 294 339) word
 
2. congruencia de_triangulos_1
2. congruencia de_triangulos_12. congruencia de_triangulos_1
2. congruencia de_triangulos_1
 
Congruencia de triangulos,modulo 2
Congruencia de triangulos,modulo 2Congruencia de triangulos,modulo 2
Congruencia de triangulos,modulo 2
 
Geometria 5° 3 b
Geometria 5° 3 bGeometria 5° 3 b
Geometria 5° 3 b
 
Congruencia de triángulos
Congruencia de triángulosCongruencia de triángulos
Congruencia de triángulos
 
S1
S1S1
S1
 
Ayudamate2015 03-25
Ayudamate2015 03-25Ayudamate2015 03-25
Ayudamate2015 03-25
 
Construcciones BáSicas Para Congruencia
Construcciones BáSicas Para CongruenciaConstrucciones BáSicas Para Congruencia
Construcciones BáSicas Para Congruencia
 
Triangulos
TriangulosTriangulos
Triangulos
 
Clase 3 triangulos II
Clase 3 triangulos IIClase 3 triangulos II
Clase 3 triangulos II
 
Ejercicios detallados del obj 10 mat i (175)
Ejercicios detallados del obj 10 mat i (175)Ejercicios detallados del obj 10 mat i (175)
Ejercicios detallados del obj 10 mat i (175)
 
GuíA De Semejanza
GuíA De SemejanzaGuíA De Semejanza
GuíA De Semejanza
 
Numero Pi
Numero PiNumero Pi
Numero Pi
 
Ejercicios resueltos yyyysemejanza de triángulos
Ejercicios resueltos yyyysemejanza de triángulosEjercicios resueltos yyyysemejanza de triángulos
Ejercicios resueltos yyyysemejanza de triángulos
 
CARLOS QUILLE RECALDE
CARLOS QUILLE RECALDECARLOS QUILLE RECALDE
CARLOS QUILLE RECALDE
 

Destacado

Unidades de almacenamiento ópticas
Unidades de almacenamiento ópticasUnidades de almacenamiento ópticas
Unidades de almacenamiento ópticasGabriela Ramirez
 
Tour du Haut Var
Tour du Haut VarTour du Haut Var
Tour du Haut VarLECREURER
 
Origen de universo
Origen de universo Origen de universo
Origen de universo 67x
 
INTERREG B - Saisir les opportunités
INTERREG B - Saisir les opportunitésINTERREG B - Saisir les opportunités
INTERREG B - Saisir les opportunitésregiosuisse
 
Les Trophées de la famille - 2015 - 2ème édition
Les Trophées de la famille - 2015 - 2ème éditionLes Trophées de la famille - 2015 - 2ème édition
Les Trophées de la famille - 2015 - 2ème éditionBarbara HADDAD
 
Planificació Centrada en la Persona: materials
Planificació Centrada en la Persona:  materialsPlanificació Centrada en la Persona:  materials
Planificació Centrada en la Persona: materialsEmili Grande
 
Trabajo practico n°9 y 10 lucila zamora
Trabajo practico n°9 y 10 lucila zamoraTrabajo practico n°9 y 10 lucila zamora
Trabajo practico n°9 y 10 lucila zamoraluly-z
 
Prospective vidéoprotection 3.0
Prospective vidéoprotection 3.0Prospective vidéoprotection 3.0
Prospective vidéoprotection 3.0Personal Interactor
 
Nouveauté De Mon RéCifal
Nouveauté De Mon RéCifalNouveauté De Mon RéCifal
Nouveauté De Mon RéCifalcedric39
 
Guide pratique CAHM-TV
Guide pratique CAHM-TVGuide pratique CAHM-TV
Guide pratique CAHM-TVlongbri
 
Assistance en ligne (version ppt 2007) - diapo
Assistance en ligne (version ppt   2007) - diapoAssistance en ligne (version ppt   2007) - diapo
Assistance en ligne (version ppt 2007) - diapoDanielle Pelchat
 

Destacado (20)

Unidades de almacenamiento ópticas
Unidades de almacenamiento ópticasUnidades de almacenamiento ópticas
Unidades de almacenamiento ópticas
 
Presentación
PresentaciónPresentación
Presentación
 
Tour du Haut Var
Tour du Haut VarTour du Haut Var
Tour du Haut Var
 
Origen de universo
Origen de universo Origen de universo
Origen de universo
 
Mapas transgenicos[1]
Mapas transgenicos[1]Mapas transgenicos[1]
Mapas transgenicos[1]
 
INTERREG B - Saisir les opportunités
INTERREG B - Saisir les opportunitésINTERREG B - Saisir les opportunités
INTERREG B - Saisir les opportunités
 
Flower
FlowerFlower
Flower
 
Open data bm
Open data bmOpen data bm
Open data bm
 
Les Trophées de la famille - 2015 - 2ème édition
Les Trophées de la famille - 2015 - 2ème éditionLes Trophées de la famille - 2015 - 2ème édition
Les Trophées de la famille - 2015 - 2ème édition
 
Planificació Centrada en la Persona: materials
Planificació Centrada en la Persona:  materialsPlanificació Centrada en la Persona:  materials
Planificació Centrada en la Persona: materials
 
Ruby en 20 minutos
Ruby en 20 minutosRuby en 20 minutos
Ruby en 20 minutos
 
L'univers Google
L'univers GoogleL'univers Google
L'univers Google
 
Trabajo practico n°9 y 10 lucila zamora
Trabajo practico n°9 y 10 lucila zamoraTrabajo practico n°9 y 10 lucila zamora
Trabajo practico n°9 y 10 lucila zamora
 
Prospective vidéoprotection 3.0
Prospective vidéoprotection 3.0Prospective vidéoprotection 3.0
Prospective vidéoprotection 3.0
 
Nouveauté De Mon RéCifal
Nouveauté De Mon RéCifalNouveauté De Mon RéCifal
Nouveauté De Mon RéCifal
 
Smart phone
Smart phoneSmart phone
Smart phone
 
Guide pratique CAHM-TV
Guide pratique CAHM-TVGuide pratique CAHM-TV
Guide pratique CAHM-TV
 
Hôtel+de+..
Hôtel+de+..Hôtel+de+..
Hôtel+de+..
 
Slide
SlideSlide
Slide
 
Assistance en ligne (version ppt 2007) - diapo
Assistance en ligne (version ppt   2007) - diapoAssistance en ligne (version ppt   2007) - diapo
Assistance en ligne (version ppt 2007) - diapo
 

Similar a Triangulos

Semejanza de triangulos
Semejanza de triangulosSemejanza de triangulos
Semejanza de triangulosNikol Narvaez
 
2 tema 9 la proporción y estructuras modulares parte 1 y 2
2 tema 9 la proporción y estructuras modulares parte 1 y 22 tema 9 la proporción y estructuras modulares parte 1 y 2
2 tema 9 la proporción y estructuras modulares parte 1 y 2qvrrafa
 
Nm2 semejanza triangulos
Nm2 semejanza triangulosNm2 semejanza triangulos
Nm2 semejanza triangulosRoberto Rojas
 
5 clase 5_proporciones_y_thales-1
5 clase 5_proporciones_y_thales-15 clase 5_proporciones_y_thales-1
5 clase 5_proporciones_y_thales-1Sebastian Valdez
 
5 clase 5_proporciones_y_thales
5 clase 5_proporciones_y_thales5 clase 5_proporciones_y_thales
5 clase 5_proporciones_y_thalesSebastian Valdez
 
thales-pitagoras-y-congruencia-de-triangulos(1).ppt
thales-pitagoras-y-congruencia-de-triangulos(1).pptthales-pitagoras-y-congruencia-de-triangulos(1).ppt
thales-pitagoras-y-congruencia-de-triangulos(1).pptAlejandro Cauich Aragon
 
Semejanza: Teorema tales
Semejanza: Teorema talesSemejanza: Teorema tales
Semejanza: Teorema talesjcremiro
 
Semejanza teorema tales
Semejanza teorema talesSemejanza teorema tales
Semejanza teorema talesjcremiro
 
Proporcionalidad de segmentos
Proporcionalidad de segmentosProporcionalidad de segmentos
Proporcionalidad de segmentosdoreligp21041969
 
Congruenciasdefigurasplanas
CongruenciasdefigurasplanasCongruenciasdefigurasplanas
Congruenciasdefigurasplanasjennifer
 
Congruencias de figuras
Congruencias de figuras Congruencias de figuras
Congruencias de figuras sitayanis
 
Precalculo de villena 05 - geometría plana
Precalculo de villena   05 - geometría planaPrecalculo de villena   05 - geometría plana
Precalculo de villena 05 - geometría planaJaime Ortiz
 

Similar a Triangulos (20)

Ma 30 2007
Ma 30 2007Ma 30 2007
Ma 30 2007
 
Ma 30 2007
Ma 30 2007Ma 30 2007
Ma 30 2007
 
Semejanza de triangulos
Semejanza de triangulosSemejanza de triangulos
Semejanza de triangulos
 
2 tema 9 la proporción y estructuras modulares parte 1 y 2
2 tema 9 la proporción y estructuras modulares parte 1 y 22 tema 9 la proporción y estructuras modulares parte 1 y 2
2 tema 9 la proporción y estructuras modulares parte 1 y 2
 
Nm2 semejanza triangulos
Nm2 semejanza triangulosNm2 semejanza triangulos
Nm2 semejanza triangulos
 
5 clase 5_proporciones_y_thales-1
5 clase 5_proporciones_y_thales-15 clase 5_proporciones_y_thales-1
5 clase 5_proporciones_y_thales-1
 
5 clase 5_proporciones_y_thales
5 clase 5_proporciones_y_thales5 clase 5_proporciones_y_thales
5 clase 5_proporciones_y_thales
 
thales-pitagoras-y-congruencia-de-triangulos(1).ppt
thales-pitagoras-y-congruencia-de-triangulos(1).pptthales-pitagoras-y-congruencia-de-triangulos(1).ppt
thales-pitagoras-y-congruencia-de-triangulos(1).ppt
 
Sesion7mf
Sesion7mfSesion7mf
Sesion7mf
 
Sesion7mf
Sesion7mfSesion7mf
Sesion7mf
 
Semejanza: Teorema tales
Semejanza: Teorema talesSemejanza: Teorema tales
Semejanza: Teorema tales
 
Semejanza teorema tales
Semejanza teorema talesSemejanza teorema tales
Semejanza teorema tales
 
Proporcionalidad de segmentos
Proporcionalidad de segmentosProporcionalidad de segmentos
Proporcionalidad de segmentos
 
Congruenciasdefigurasplanas
CongruenciasdefigurasplanasCongruenciasdefigurasplanas
Congruenciasdefigurasplanas
 
Capitulo 1geometria
Capitulo 1geometriaCapitulo 1geometria
Capitulo 1geometria
 
Proporciones y semejanza
Proporciones y semejanzaProporciones y semejanza
Proporciones y semejanza
 
Congruencias de figuras
Congruencias de figuras Congruencias de figuras
Congruencias de figuras
 
Precalculo de villena 05 - geometría plana
Precalculo de villena   05 - geometría planaPrecalculo de villena   05 - geometría plana
Precalculo de villena 05 - geometría plana
 
Gep7 geometria 2013
Gep7 geometria 2013Gep7 geometria 2013
Gep7 geometria 2013
 
Triángulos
TriángulosTriángulos
Triángulos
 

Más de Irvingg Kennedy Mcfly (20)

Efectos de la música multidimensional
Efectos de la música multidimensionalEfectos de la música multidimensional
Efectos de la música multidimensional
 
Ciencia e ingenieria resumen
Ciencia e ingenieria resumenCiencia e ingenieria resumen
Ciencia e ingenieria resumen
 
Practica2 Algoritmo de cambio de base, de la division
Practica2 Algoritmo de cambio de base, de la divisionPractica2 Algoritmo de cambio de base, de la division
Practica2 Algoritmo de cambio de base, de la division
 
Transmisión de datos progra
Transmisión de datos prograTransmisión de datos progra
Transmisión de datos progra
 
Reseña critica de rayuela de julio cortázar
Reseña critica de rayuela de julio cortázarReseña critica de rayuela de julio cortázar
Reseña critica de rayuela de julio cortázar
 
Formulario de tiempos en inglés
Formulario de tiempos en inglésFormulario de tiempos en inglés
Formulario de tiempos en inglés
 
Charles Darwin and his Theory
Charles Darwin and his TheoryCharles Darwin and his Theory
Charles Darwin and his Theory
 
Corriente continua y corriente alterna
Corriente continua y corriente alternaCorriente continua y corriente alterna
Corriente continua y corriente alterna
 
Leng prog C
Leng prog CLeng prog C
Leng prog C
 
Algebra lineal y numeros complejos
Algebra lineal y numeros complejosAlgebra lineal y numeros complejos
Algebra lineal y numeros complejos
 
áCido úrico
áCido úricoáCido úrico
áCido úrico
 
Nuestra salud
Nuestra saludNuestra salud
Nuestra salud
 
Pitagoras
PitagorasPitagoras
Pitagoras
 
Leyes de kirchhoff
Leyes de kirchhoffLeyes de kirchhoff
Leyes de kirchhoff
 
Leyes de k
Leyes de kLeyes de k
Leyes de k
 
Dango dango dango dango dango daikazoku
Dango   dango   dango   dango   dango   daikazokuDango   dango   dango   dango   dango   daikazoku
Dango dango dango dango dango daikazoku
 
Hardware
HardwareHardware
Hardware
 
Galva
GalvaGalva
Galva
 
6127813 leyes-de-kirchhoff
6127813 leyes-de-kirchhoff6127813 leyes-de-kirchhoff
6127813 leyes-de-kirchhoff
 
555 Flipflop
555 Flipflop555 Flipflop
555 Flipflop
 

Último

TUTORIA II - CIRCULO DORADO UNIVERSIDAD CESAR VALLEJO
TUTORIA II - CIRCULO DORADO UNIVERSIDAD CESAR VALLEJOTUTORIA II - CIRCULO DORADO UNIVERSIDAD CESAR VALLEJO
TUTORIA II - CIRCULO DORADO UNIVERSIDAD CESAR VALLEJOweislaco
 
c3.hu3.p1.p3.El ser humano como ser histórico.pptx
c3.hu3.p1.p3.El ser humano como ser histórico.pptxc3.hu3.p1.p3.El ser humano como ser histórico.pptx
c3.hu3.p1.p3.El ser humano como ser histórico.pptxMartín Ramírez
 
RAIZ CUADRADA Y CUBICA PARA NIÑOS DE PRIMARIA
RAIZ CUADRADA Y CUBICA PARA NIÑOS DE PRIMARIARAIZ CUADRADA Y CUBICA PARA NIÑOS DE PRIMARIA
RAIZ CUADRADA Y CUBICA PARA NIÑOS DE PRIMARIACarlos Campaña Montenegro
 
Análisis de la Implementación de los Servicios Locales de Educación Pública p...
Análisis de la Implementación de los Servicios Locales de Educación Pública p...Análisis de la Implementación de los Servicios Locales de Educación Pública p...
Análisis de la Implementación de los Servicios Locales de Educación Pública p...Baker Publishing Company
 
La Función tecnológica del tutor.pptx
La  Función  tecnológica  del tutor.pptxLa  Función  tecnológica  del tutor.pptx
La Función tecnológica del tutor.pptxJunkotantik
 
el CTE 6 DOCENTES 2 2023-2024abcdefghijoklmnñopqrstuvwxyz
el CTE 6 DOCENTES 2 2023-2024abcdefghijoklmnñopqrstuvwxyzel CTE 6 DOCENTES 2 2023-2024abcdefghijoklmnñopqrstuvwxyz
el CTE 6 DOCENTES 2 2023-2024abcdefghijoklmnñopqrstuvwxyzprofefilete
 
Estrategias de enseñanza - aprendizaje. Seminario de Tecnologia..pptx.pdf
Estrategias de enseñanza - aprendizaje. Seminario de Tecnologia..pptx.pdfEstrategias de enseñanza - aprendizaje. Seminario de Tecnologia..pptx.pdf
Estrategias de enseñanza - aprendizaje. Seminario de Tecnologia..pptx.pdfAlfredoRamirez953210
 
TEST DE RAVEN es un test conocido para la personalidad.pdf
TEST DE RAVEN es un test conocido para la personalidad.pdfTEST DE RAVEN es un test conocido para la personalidad.pdf
TEST DE RAVEN es un test conocido para la personalidad.pdfDannyTola1
 
CIENCIAS NATURALES 4 TO ambientes .docx
CIENCIAS NATURALES 4 TO  ambientes .docxCIENCIAS NATURALES 4 TO  ambientes .docx
CIENCIAS NATURALES 4 TO ambientes .docxAgustinaNuez21
 
Día de la Madre Tierra-1.pdf día mundial
Día de la Madre Tierra-1.pdf día mundialDía de la Madre Tierra-1.pdf día mundial
Día de la Madre Tierra-1.pdf día mundialpatriciaines1993
 
Estrategia de Enseñanza y Aprendizaje.pdf
Estrategia de Enseñanza y Aprendizaje.pdfEstrategia de Enseñanza y Aprendizaje.pdf
Estrategia de Enseñanza y Aprendizaje.pdfromanmillans
 
OLIMPIADA DEL CONOCIMIENTO INFANTIL 2024.pptx
OLIMPIADA DEL CONOCIMIENTO INFANTIL 2024.pptxOLIMPIADA DEL CONOCIMIENTO INFANTIL 2024.pptx
OLIMPIADA DEL CONOCIMIENTO INFANTIL 2024.pptxjosetrinidadchavez
 
FICHA DE MONITOREO Y ACOMPAÑAMIENTO 2024 MINEDU
FICHA DE MONITOREO Y ACOMPAÑAMIENTO  2024 MINEDUFICHA DE MONITOREO Y ACOMPAÑAMIENTO  2024 MINEDU
FICHA DE MONITOREO Y ACOMPAÑAMIENTO 2024 MINEDUgustavorojas179704
 
PPT GESTIÓN ESCOLAR 2024 Comités y Compromisos.pptx
PPT GESTIÓN ESCOLAR 2024 Comités y Compromisos.pptxPPT GESTIÓN ESCOLAR 2024 Comités y Compromisos.pptx
PPT GESTIÓN ESCOLAR 2024 Comités y Compromisos.pptxOscarEduardoSanchezC
 
BIOLOGIA_banco de preguntas_editorial icfes examen de estado .pdf
BIOLOGIA_banco de preguntas_editorial icfes examen de estado .pdfBIOLOGIA_banco de preguntas_editorial icfes examen de estado .pdf
BIOLOGIA_banco de preguntas_editorial icfes examen de estado .pdfCESARMALAGA4
 
TRIPTICO-SISTEMA-MUSCULAR. PARA NIÑOS DE PRIMARIA
TRIPTICO-SISTEMA-MUSCULAR. PARA NIÑOS DE PRIMARIATRIPTICO-SISTEMA-MUSCULAR. PARA NIÑOS DE PRIMARIA
TRIPTICO-SISTEMA-MUSCULAR. PARA NIÑOS DE PRIMARIAAbelardoVelaAlbrecht1
 

Último (20)

DIA INTERNACIONAL DAS FLORESTAS .
DIA INTERNACIONAL DAS FLORESTAS         .DIA INTERNACIONAL DAS FLORESTAS         .
DIA INTERNACIONAL DAS FLORESTAS .
 
TUTORIA II - CIRCULO DORADO UNIVERSIDAD CESAR VALLEJO
TUTORIA II - CIRCULO DORADO UNIVERSIDAD CESAR VALLEJOTUTORIA II - CIRCULO DORADO UNIVERSIDAD CESAR VALLEJO
TUTORIA II - CIRCULO DORADO UNIVERSIDAD CESAR VALLEJO
 
c3.hu3.p1.p3.El ser humano como ser histórico.pptx
c3.hu3.p1.p3.El ser humano como ser histórico.pptxc3.hu3.p1.p3.El ser humano como ser histórico.pptx
c3.hu3.p1.p3.El ser humano como ser histórico.pptx
 
RAIZ CUADRADA Y CUBICA PARA NIÑOS DE PRIMARIA
RAIZ CUADRADA Y CUBICA PARA NIÑOS DE PRIMARIARAIZ CUADRADA Y CUBICA PARA NIÑOS DE PRIMARIA
RAIZ CUADRADA Y CUBICA PARA NIÑOS DE PRIMARIA
 
Análisis de la Implementación de los Servicios Locales de Educación Pública p...
Análisis de la Implementación de los Servicios Locales de Educación Pública p...Análisis de la Implementación de los Servicios Locales de Educación Pública p...
Análisis de la Implementación de los Servicios Locales de Educación Pública p...
 
La Función tecnológica del tutor.pptx
La  Función  tecnológica  del tutor.pptxLa  Función  tecnológica  del tutor.pptx
La Función tecnológica del tutor.pptx
 
el CTE 6 DOCENTES 2 2023-2024abcdefghijoklmnñopqrstuvwxyz
el CTE 6 DOCENTES 2 2023-2024abcdefghijoklmnñopqrstuvwxyzel CTE 6 DOCENTES 2 2023-2024abcdefghijoklmnñopqrstuvwxyz
el CTE 6 DOCENTES 2 2023-2024abcdefghijoklmnñopqrstuvwxyz
 
Estrategias de enseñanza - aprendizaje. Seminario de Tecnologia..pptx.pdf
Estrategias de enseñanza - aprendizaje. Seminario de Tecnologia..pptx.pdfEstrategias de enseñanza - aprendizaje. Seminario de Tecnologia..pptx.pdf
Estrategias de enseñanza - aprendizaje. Seminario de Tecnologia..pptx.pdf
 
Unidad 3 | Teorías de la Comunicación | MCDI
Unidad 3 | Teorías de la Comunicación | MCDIUnidad 3 | Teorías de la Comunicación | MCDI
Unidad 3 | Teorías de la Comunicación | MCDI
 
TEST DE RAVEN es un test conocido para la personalidad.pdf
TEST DE RAVEN es un test conocido para la personalidad.pdfTEST DE RAVEN es un test conocido para la personalidad.pdf
TEST DE RAVEN es un test conocido para la personalidad.pdf
 
CIENCIAS NATURALES 4 TO ambientes .docx
CIENCIAS NATURALES 4 TO  ambientes .docxCIENCIAS NATURALES 4 TO  ambientes .docx
CIENCIAS NATURALES 4 TO ambientes .docx
 
Día de la Madre Tierra-1.pdf día mundial
Día de la Madre Tierra-1.pdf día mundialDía de la Madre Tierra-1.pdf día mundial
Día de la Madre Tierra-1.pdf día mundial
 
Estrategia de Enseñanza y Aprendizaje.pdf
Estrategia de Enseñanza y Aprendizaje.pdfEstrategia de Enseñanza y Aprendizaje.pdf
Estrategia de Enseñanza y Aprendizaje.pdf
 
OLIMPIADA DEL CONOCIMIENTO INFANTIL 2024.pptx
OLIMPIADA DEL CONOCIMIENTO INFANTIL 2024.pptxOLIMPIADA DEL CONOCIMIENTO INFANTIL 2024.pptx
OLIMPIADA DEL CONOCIMIENTO INFANTIL 2024.pptx
 
FICHA DE MONITOREO Y ACOMPAÑAMIENTO 2024 MINEDU
FICHA DE MONITOREO Y ACOMPAÑAMIENTO  2024 MINEDUFICHA DE MONITOREO Y ACOMPAÑAMIENTO  2024 MINEDU
FICHA DE MONITOREO Y ACOMPAÑAMIENTO 2024 MINEDU
 
PPT GESTIÓN ESCOLAR 2024 Comités y Compromisos.pptx
PPT GESTIÓN ESCOLAR 2024 Comités y Compromisos.pptxPPT GESTIÓN ESCOLAR 2024 Comités y Compromisos.pptx
PPT GESTIÓN ESCOLAR 2024 Comités y Compromisos.pptx
 
BIOLOGIA_banco de preguntas_editorial icfes examen de estado .pdf
BIOLOGIA_banco de preguntas_editorial icfes examen de estado .pdfBIOLOGIA_banco de preguntas_editorial icfes examen de estado .pdf
BIOLOGIA_banco de preguntas_editorial icfes examen de estado .pdf
 
VISITA À PROTEÇÃO CIVIL _
VISITA À PROTEÇÃO CIVIL                  _VISITA À PROTEÇÃO CIVIL                  _
VISITA À PROTEÇÃO CIVIL _
 
Earth Day Everyday 2024 54th anniversary
Earth Day Everyday 2024 54th anniversaryEarth Day Everyday 2024 54th anniversary
Earth Day Everyday 2024 54th anniversary
 
TRIPTICO-SISTEMA-MUSCULAR. PARA NIÑOS DE PRIMARIA
TRIPTICO-SISTEMA-MUSCULAR. PARA NIÑOS DE PRIMARIATRIPTICO-SISTEMA-MUSCULAR. PARA NIÑOS DE PRIMARIA
TRIPTICO-SISTEMA-MUSCULAR. PARA NIÑOS DE PRIMARIA
 

Triangulos

  • 1. atic as CAP´ ITULO 7 eM atem SEMEJANZA ept o. d ´ INTRODUCCION 7.1. a, D Definici´n 1. o a. Raz´n: se llama raz´n, al cociente de dos cantidades, o o expresadas en la misma magnitud, por ejemplo a . b dad de An tio qui b. Proporci´n: se llama proporci´n a la igualdad de dos razones. Por o o c e e ejemplo a = d , a los t´rminos a y d se les llama extremos y los t´rminos b ¯ ¯ b y c se les llama medios, al t´rmino d se le llama cuarta proporcional e ¯ ¯ ¯ entre a, b y c en este orden. ¯ ¯ ¯ En algunos textos de geometr´ se utiliza la notaci´n de proporci´n as´ ıa o o ı: a : b = c : d que se lee “a es a b como c es a d” ¯ ¯ ¯ ¯ ersi Propiedades de las proporciones: a b = c d entonces a · d = b · c 2. Si a b = c d y 3. Si a b = c d entonces b a 4. Si a b = c d entonces a±b b 5. Si a b = c d a b = c e Un iv 1. Si entonces d = e entonces = d c = a+b c+d o a c c±d d = = b d o a±b a a−b c−d o o d b = a+b a−b 201 = c a c±d c = c+d c−d
  • 2. CAP´ ITULO 7. SEMEJANZA, JAIME ESCOBAR A. 202 6. Si a1 b1 = a2 b2 = a3 b3 = ... = an bn entonces a2 a3 an a1 + a 2 a1 + a 2 + . . . + a n a1 = = = ... = = = ... = b1 b2 b3 bn b1 + b 2 b1 + b 2 + . . . + b n atic 7.2. as b 7. Si b es una magnitud tal que a = d , entonces decimos que b es media b ¯ ¯ proporcional entre a y d o lo que es lo mismo: b es media proporcional ¯ ¯2 ¯ entre a y d si y solo si b = a · d. ¯ ¯ PARALELISMO Y PROPORCIONALIDAD atem ←→ B o. d P   A eM o Definici´n 2. o 1. Un punto P ∈AB divide al segmento AB en una raz´n PA r si P B = r. Si r = 1 entonces P es el punto medio de AB.     a, D ept Figura 1. ←→ ←→ An tio qui 2. Sean AB y CD y sean X ∈AB y Y ∈CD, decimos que X e Y dividen a AB y CD en segmentos proporcionales si A X ¡ Y dad ¡ D ¡ ¡ Un iv ersi ¡ C B ¡ de XA YC = XB YD Figura 2. El siguiente Lema, llamado el Teorema fundamental del paralelismo es en realidad una generalizaci´n del Teorema de la paralela media. o
  • 3. 7.2. PARALELISMO Y PROPORCIONALIDAD 203 Lema 1 (Teorema fundamental del paralelismo). Si tres o m´s rectas paralelas determinan segmentos congruentes en una a secante entonces determinan segmentos congruentes sobre cualquier otra secante. E F m C eM E’ G n o. d F’ H a, D b An tio Figura 3. r qui G’ ept D a atic B l atem A as Demostraci´n. (Ver Figura 3.) o ←→ de Sean l, m, n, r cuatro rectas paralelas y a una secante que corta a estas paralelas en A, B, C, D tales que AB ∼ BC ∼ CD. Sea b otra secante que = = corta a las paralelas en E, F, G, H. Veamos que EF ∼ F G ∼ GH. = = ←→ ←→ Un iv ersi dad Por Playfair, por E, F, G pasan EE , F F , GG paralelas a a, donde E ∈ m, F ∈ n, G ∈ r. Por la proposici´n 2., AB ∼ EE , BC ∼ F F y CD ∼ GG luego EE ∼ o = = = = ∼ GG y como los angulos E EF ∼ F F G ∼ G GH por correspondientes FF = ´ = = entre paralelas y por la proposici´n n´mero 1, EE F ∼ F F G ∼ GG H y o u = = por el criterio A-L-A, los siguientes tri´ngulos son congruentes: a EE F ∼ = FF G ∼ = luego EF ∼ F G ∼ GH = = GG H,
  • 4. CAP´ ITULO 7. SEMEJANZA, JAIME ESCOBAR A. 204 Teorema 1. Dado un n´mero entero n y dado un segmento, existen puntos en el interior u del segmento que lo dividen en n segmentos congruentes. as An−1 An l ¢ ¢ atem A2 Figura 4. atic ¢ A1 ¢ A P2 ¢ P1 Pn B ¢ Pn−1 o. d eM Demostraci´n. (Ver Figura 4.) o Sea AB un segmento y n un n´mero entero, veamos que existen puntos P u que dividen al segmento en n segmentos congruentes. Sea l una semirrecta ←→ a, D ept cualesquiera, con origen en A tal que l no est´ contenida en la recta AB. e ımedes, existen Sobre la semirrecta l, por el Axioma de continuidad de Arqu´ puntos A1 , A2 , . . . , An−1 , An tales que qui AA1 ∼ A1 A2 ∼ . . . An−1 An = = An tio Por Playfair, por A1 , A2 , . . . , An−1 pasan paralelas a An B, las cuales se intersectan con AB en P1 , P2 , . . . , Pn−1 , entonces por el lema anterior de AP1 ∼ P1 P2 ∼ . . . Pn−1 B = = Un iv ersi dad Definici´n 3 (Segmentos conmensurables e inconmensurables). Deo cimos que un segmento es conmensurable si su medida es un n´mero racional u y decimos que un segmento es inconmensurable si su medida es un n´mero u irracional. Teorema 2 (Teorema de Tales). Si tres o m´s paralelas cortan a dos o m´s secantes entonces los segmentos a a que determinan en ellas son proporcionales. Demostraci´n. (Ver Figura 5.) o ←→ ←→ ←→ Sean AD, BE y CF rectas paralelas que cortan las secantes a, b en los puntos
  • 5. 7.2. PARALELISMO Y PROPORCIONALIDAD A 205 D D1 A1 D2 A2 D3 A3 An−1 Dn−1 B E as E1 B1 Em Bm F Em+1 a eM b atem C Bm+1 atic E2 B2 ept o. d Figura 5. de An tio qui a, D A, D, B, E, C, F respectivamente. AB EF Veamos que BC = DE o equivalentemente BC = DE EF AB BC EF Llamemos x = AB e y = DE y veamos que x = y Sea n un n´mero entero cualesquiera, entonces por el Teorema 1., existen u puntos A1 , A2 , . . . , An−1 que dividen al segmento AB en n segmentos congruentes: AA1 ∼ A1 A2 ∼ A2 A3 ∼ . . . ∼ An−1 B, AB = nAA1 . = = = = ←→ ersi dad Por Playfair, por A1 , A2 , . . . , An−1 pasan rectas paralelas a AD que cortan a b en D1 , D2 , . . . , Dn−1 , luego por el Lema 1. (Teorema fundamental del paralelismo), los segmentos: DD1 ∼ D1 D2 ∼ D2 D3 ∼ . . . ∼ Dn−1 E, DE = nDD1 . = = = = Un iv Por el Axioma de Arqu´ ımedes, existen puntos B1 , B2 , . . . , Bm , Bm+1 en la − − → BC tales que BB1 ∼ B1 B2 ∼ B2 B3 ∼ . . . ∼ Bm Bm+1 ∼ AA1 , = = = = = y C entre B, Bm+1 , por tanto BBm = mAA1 . Luego, BBm mAA1 m = = AB nAA1 n
  • 6. CAP´ ITULO 7. SEMEJANZA, JAIME ESCOBAR A. 206 ←→ Por Playfair, por B1 , B2 , . . . , Bm , Bm+1 pasan paralelas a AD que cortan a − → EF en los puntos E1 , E2 , . . . , Em , Em+1 y por el Lema 1. (Teorema fundamental del paralelismo), EE1 ∼ E1 E2 ∼ E2 E3 ∼ . . . ∼ Em Em+1 , = = = = y F entre E y Em+1 , ya que C esta entre B y Bm+1 luego o b.) x > atem m n m . n eM Dos casos pueden ocurrir: a.) x = a.) Si x = m , entonces n atic EEm mDD1 m = = DE nDD1 n as EEm = mDD1 . ept o. d m mAA1 BBm BC =x= = = AB n nAA1 AB BC m > nAA1 n m n entonces x = BC AB > m n o sea que An tio b.) Supongamos que x > qui a, D y por lo tanto BC = BBm y como C y Bm est´n del mismo lado con respecto a a B entonces por el axioma de construcci´n de segmento, C ≡ Bm , entonces o EF F ≡ Em , luego m = EEm = DE = y. n DE y mAA1 < BC < (m + 1)AA1 de y dad mDD1 < EF < (m + 1)DD1 por lo tanto Un iv ersi EF mDD1 m EF = > = . DE nDD1 nDD1 n En resumen, hemos demostrado que si x > m entonces y > m . n n De la misma manera se demuestra que si y > m entonces x > m . n n Hasta aqu´ hemos demostrado que para todo n´ mero racional m , si ı, u n x > m entonces y > m y rec´ ıprocamente, si y > m entonces x > m . En n n n n otras palabras, todo n´mero racional a la izquierda de x esta tambi´n a la u e izquierda de y y todo n´mero racional a la izquierda de y esta a la izquierda u de x. Todo esto significa que no hay un n´ mero racional entre x e y, ya u y=
  • 7. 7.2. PARALELISMO Y PROPORCIONALIDAD 207 que si hubiera un n´mero racional entre x e y entonces estar´ a la izquierda u ıa de uno de ellos y a la derecha del otro, lo cual contradice lo demostrado; por lo tanto x = y, es decir: AB DE = BC EF as Corolario 1 (Teorema de Tales en el tri´ngulo). Toda recta paralela a a un lado de un tri´ngulo y que corte a los otros dos lados, divide a estos lados a en segmentos proporcionales. eM atem atic A F o. d E C a, D qui Figura 6. ept B luego y = FA ; FC por EA FA = AE + EB AF + F C de FA + FC EA + EB = EB FC EA EB An tio Lo que afirma este corolario es que si EF ||BC entonces las propiedades de las fracciones AB AC = AE AF Un iv ersi dad AB AC = y EB FC esto demuestra el siguiente corolario. Corolario 2. Dos lados de un tri´ngulo son proporcionales a los segmentos a que en ellos determina cualquier recta paralela al tercer lado. El siguiente teorema es el rec´ ıproco del Corolario 1 Teorema 3 (Rec´ ıproco del Teorema de Tales en el tri´ngulo). a Si una recta intercepta dos lados de un tri´ngulo en segmentos propora cionales entonces la recta es paralela al tercer lado del tri´ngulo. a
  • 8. CAP´ ITULO 7. SEMEJANZA, JAIME ESCOBAR A. 208 A F l E F’ C atem atic Figura 7. as B l AE AF = EB FC o. d l ∩ AB = {E}, l ∩ AC = {F }, eM Demostraci´n. (Ver Figura 7.) o Sea el ABC y l una recta tal que AF FC = AF F C An tio luego qui AE AF = EB FC a, D ept Por Playfair, por E pasa l ||BC la cual intercepta a AC en F , entonces por el Corolario 1 (Teorema de Tales en el tri´ngulo), se tiene: a y por las propiedades de las fracciones FC AF = F C AF o sea que dad que es lo mismo que de F C + AF F C + AF = AF AF Un iv ersi AC AC = AF AF luego AF = AF y por tanto AF ∼ AF y como F, F est´n del mismo lado a = con respecto a A entonces por el axioma de construcci´n de segmento F ≡ F o y por lo tanto EF ||BC. En forma similar se demuestran los siguientes rec´ ıprocos: Corolario 3 (Rec´ ıproco del Corolario 2). Si dos lados de un tri´ngulo son a proporcionales a los segmentos que en ella determina una recta que intercepta los dos lados, entonces la recta es paralela al tercer lado del tri´ngulo. a
  • 9. 7.2. PARALELISMO Y PROPORCIONALIDAD 209 A E F C AB AC = EB FC o ´ o. d AB AC = AE AF ABC (Ver Figura 8.) eM Lo que afirma este corolario es que si en el atem atic Figura 8. as B a, D ept entonces EF ||BC An tio qui Teorema 4 (Propiedades m´tricas de la bisectriz de un tri´ngulo). e a La bisectriz de un angulo de un tri´ngulo divide al lado opuesto en segmen´ a tos proporcionales a los otros dos lados. Demostraci´n. (Ver Figura 9.) o Sea AV bisectriz de A en el ABC con V ∈ IntBC. Veamos que ←→ VB VC = AB . AC Un iv ersi dad de Por Playfair, por C pasa l||AV ; sea {D} = l∩ BA, luego por alternos internos entre paralelas, V AC ∼ ACD y por correspondientes entre par= ∼ ADC, pero como AV es bisectriz por hip´tesis, entonces alelas, BAV = o ∼ V AC, luego BAV = ADC ∼ ACD = y por el teorema del tri´ngulo is´sceles, se tiene que a o por lo tanto AD ∼ AC. = Por el corolario 2 (Teorema de Tales en el tri´ngulo), a AB VB = , VC AD ADC es is´sceles y o
  • 10. CAP´ ITULO 7. SEMEJANZA, JAIME ESCOBAR A. 210 l D o. d C ept V Figura 9. a, D B eM atem atic as A qui luego An tio AB VB = . VC AC ersi dad de Teorema 5 (Rec´ ıproco del teorema anterior). Si una recta que pasa por el v´rtice de un tri´ngulo divide al lado opuesto e a en segmentos proporcionales a los otros dos lados, entonces esta recta es bisectriz del angulo ubicado en el v´rtice por donde pasa la recta. ´ e Un iv Demostraci´n. (Ver Figura 10.) o ←→ Supongamos que en el ABC se tiene que AV con V ∈ IntBC, tal que VB = AB . Veamos que AV es bisectriz de A. VC AC ←→ Por Playfair, por C pasa l||AV ; sea {D} = l∩ BA. AB Como l||AV , entonces por el corolario 2, V B = AD , pero por hip´tesis o VC AB VB = , VC AC
  • 11. 7.2. PARALELISMO Y PROPORCIONALIDAD 211 l D o. d C ept V Figura 10. a, D B eM atem atic as A entonces ersi dad de An tio qui AB AB = AC AD y por las propiedades de las fracciones AD = AC o sea que AD ∼ AC, = por lo tanto el ADC es is´sceles y por el Teorema del tri´ngulo is´sceles, o a o ADC ∼ ACD. = Por otro lado, por alternos internos entre paralelas, V AC ∼ ACD y por = ∼ ADC. correspondientes entre paralelas, BAV = Luego BAV ∼ V AC, luego AV es bisectriz de A. = Un iv Teorema 6 (Propiedades m´tricas de la bisectriz exterior de un e tri´ngulo). a La bisectriz de un angulo exterior de un tri´ngulo, que no sea paralela al lado ´ a opuesto, divide exteriormente al lado opuesto en segmentos proporcionales a los otros dos lados. Demostraci´n. (Ver Figura 11.) o Sea AV bisectriz del angulo exterior EAC en el ´ ←→ ABC con V ∈BC y
  • 12. CAP´ ITULO 7. SEMEJANZA, JAIME ESCOBAR A. 212 E A l C B − C − V . Veamos que V B V C = V’ atem Figura 11. eM B atic as D AB . AC ←→ qui a, D ept o. d Por Playfair, por C pasa l||AV ; sea {D} = l∩ BA, luego por alternos internos entre paralelas, V AC ∼ ACD y por correspondientes entre pa= ∼ ADC, pero como AV es bisectriz por hip´tesis, entonces ralelas, EAV = o ∼ V AE, luego CAV = ADC ∼ ACD = ADC es is´sceles y o An tio y por el teorema del tri´ngulo is´sceles, se tiene que a o por lo tanto AD ∼ AC. = de Por el corolario 2 (Teorema de Tales en el tri´ngulo), a ersi luego dad AB V B = , V C AD Un iv V B AB = . V C AC El rec´ ıproco de este teorema se deja como ejercicio. Teorema 7 (Rec´ ıproco del Teorema anterior). Una recta que pase por el v´rtice de un tri´ngulo y divida la prolongaci´n del e a o lado opuesto en segmentos proporcionales a los otros dos lados del tri´ngulo, a es bisectriz del angulo exterior ubicado en este v´rtice. ´ e
  • 13. 7.3. SEMEJANZA DE POL´ IGONOS 213 Definici´n 4 (Divisi´n arm´nica). Si A y B son dos puntos distintos y o o o ←→ C ∈ IntAB y D ∈AB pero D ∈ AB, decimos que C, D dividen arm´nica/ o mente a AB si DA CA = CB DB A C B D £ £ £ £ atic as Figura 12. ept o. d eM atem A los puntos C y D se les llama los conjugados arm´nicos con respecto a o A y B. Los puntos A, B, C, D en este orden, se dice que forman una divisi´n arm´nica. o o Tambi´n, de acuerdo a la definici´n, podemos afirmar que A y B son conjue o gados arm´nicos con respecto a CD. o Por los teoremas 4 y 6 y por la definici´n de conjugado arm´nico, podemos o o afirmar el siguiente teorema. qui a, D Teorema 8. La bisectriz de un angulo de un tri´ngulo y la bisectriz del angulo exterior ´ a ´ suplementario, dividen al lado opuesto arm´nicamente. o SEMEJANZA DE POL´ IGONOS dad 7.3. de An tio Nota: de acuerdo a los teoremas anteriores, el lugar geom´trico de los puntos e A tales que la raz´n de las distancias a dos puntos fijos B y C sea una o constante k, es una circunferencia de di´metro V V , donde V, V son los a o conjugado arm´nicos de BC con raz´n k. o Un iv ersi Definici´n 5 (Pol´ o ıgonos semejantes). Decimos que dos pol´ ıgonos son semejantes si se puede establecer una correspondencia entre sus lados y sus angulos de tal manera que: ´ 1. Los lados correspondientes son proporcionales. A estos lados tambi´n los e llamaremos lados hom´logos. La raz´n r entre los lados hom´logos la llao o o mamos raz´n de semejanza. o 2. Los angulos correspondientes son congruentes. A los angulos correspon´ ´ dientes congruentes, tambi´n se les llama angulos hom´logos. e ´ o En particular, para los tri´ngulos tenemos la siguiente definici´n. a o
  • 14. CAP´ ITULO 7. SEMEJANZA, JAIME ESCOBAR A. 214 Definici´n 6 (Tri´ngulos semejantes). Decimos que el ABC es semeo a jante al A B C , lo cual denotamos as´ ABC ∼ A B C , si: ı as BC AC AB = = (∗) AB BC AC A ∼ A , B ∼ B , C ∼ C (∗∗) = = = A atem eM B’ o. d C a a C’ ept B b c b c atic A’ a, D Figura 13. An tio qui Nota: 1. Con los teoremas que haremos m´s adelante, mostraremos que (*) a implica (**) y rec´ ıprocamente, (**) implica (*). 2. Por las propiedades de las fracciones, se puede demostrar que si dos tri´ngulos son semejantes, entonces sus lados son entre si como sus per´ a ımetros, es decir, si ABC ∼ A B C entonces dad de a b c p = = = =r a b c p Un iv ersi donde p = a + b + c =per´ ımetro del ABC, p = a + b + c =per´ ımetro del A B C y r es la raz´n de semejanza. o 3. La relaci´n de semejanza entre pol´ o ıgonos es una relaci´n de equivalencia, o es decir, es reflexiva, sim´trica y transitiva (Ejercicio). e Definici´n 7 (Pol´ o ıgonos congruentes). Decimos que dos pol´ ıgonos semejantes, son congruentes si tienen sus lados hom´logos congruentes. o Teorema 9. Dos pol´ ıgonos semejantes son congruentes si un lado de uno de ellos es congruente con su hom´logo. o
  • 15. 7.3. SEMEJANZA DE POL´ IGONOS 215 A continuaci´n veremos tres criterios de semejanza de tri´ngulos. o a Teorema 10 (Primer criterio de semejanza: Angulo-Angulo (A-A)). Si dos angulos de un tri´ngulo son congruentes con dos angulos de otro ´ a ´ tri´ngulo, entonces los dos tri´ngulos son semejantes. a a A atic as A’ F C B’ C’ ept o. d B E eM D atem l a, D Figura 14. o sea Un iv AB AC = , AD AE ersi dad de An tio qui Demostraci´n. (Ver Figura 14.) Supongamos que en los tri´ngulos ABC o a ∼ B , C ∼ C , entonces por el teorema de la suma y A B C se tiene que B = = de los angulos interiores de un tri´ngulo, A ∼ A . ´ a = − → Por el axioma de construcci´n de segmento, existe un punto D ∈ AB y o − → E ∈ AC tales que AD ∼ A B y AE ∼ A C ; unamos D con E, entonces = = por el criterio L-A-L, el ADE ∼ A B C , por lo tanto DE ∼ B C , = = ADE ∼ B , pero por hip´tesis B ∼ B, por lo tanto ADE ∼ B y por el o = = = teorema de alternos internos (Teorema 31), DE||BC y por el corolario 2, ←→ AB AC = AB AC (∗) ←→ Por Playfair, por D pasa l|| AC, sea {F } = l∩ BC y por la proposici´n o ∼ F C y como DE ∼ B C entonces F C ∼ B C ; por otro n´mero 2, DE = u = = lado, por el corolario 2, BC AB = , AD FC o sea AB BC = AB BC (∗∗)
  • 16. CAP´ ITULO 7. SEMEJANZA, JAIME ESCOBAR A. 216 de (*), (**) AB AC BC = = , AB AC BC hemos mostrado que los tres pares de angulos son congruentes y los tres pares ´ de lados respectivos son proporcionales, por lo tanto ABC ∼ ABC as Se deja como ejercicio los siguientes corolarios. atem atic Corolario 4 (Paralela a un lado de un tri´ngulo). Una paralela a un a lado de un tri´ngulo determina otro tri´ngulo semejante al primero. a a eM Corolario 5. Si dos tri´ngulos rect´ngulos tienen un par de angulos agudos a a ´ respectivamente congruentes, entonces son semejantes. o. d Corolario 6. Si dos tri´ngulos tienen sus lados respectivamente paralelos o a respectivamente perpendiculares, entonces los dos tri´ngulos son semejantes. a a, D ept Corolario 7. Las alturas y las bisectrices hom´logas de dos tri´ngulos seo a mejantes est´n en la misma raz´n que sus lados hom´logos. a o o qui Corolario 8. Dos tri´ngulos is´sceles son semejantes si tienen un par de a o angulos congruentes. ´ An tio Corolario 9. Todos los tri´ngulos equil´teros son semejantes. a a ersi dad de Teorema 11 (Segundo criterio de semejanza: P-A-P). Si un angulo de un tri´ngulo es congruente con otro angulo de otro tri´ngulo ´ a ´ a y los lados que comprenden al angulo en el primer tri´ngulo son respecti´ a vamente proporcionales a los lados que comprende al angulo en el segundo ´ tri´ngulo, entonces los dos tri´ngulos son semejantes. a a Un iv Demostraci´n. (Ver figura 15.) Tomemos por hip´tesis que A ∼ A y o o = AB AC = A C . Veamos que ABC ∼ A B C . AB − → − → Por el axioma de construcci´n de segmento, existen D ∈ AB y E ∈ AC o tales que AD ∼ A B y AE ∼ A C , por lo tanto, por el criterio L-A-L, = = ∼ ABC . ADE = AB AC AC AB Por otro lado, como A B = A C entonces AD = AE y por el corolario 3 (rec´ ıproco del corolario 2), DE||BC
  • 17. 7.3. SEMEJANZA DE POL´ IGONOS 217 A A’ E B C B’ C’ as D atem ADE ∼ ABC ∼ ABC y por transitividad eM Por lo tanto, por el corolario 4, atic Figura 15. o. d ABC ept Corolario 10. Dos tri´ngulos rect´ngulos son semejantes si sus catetos son a a respectivamente proporcionales. qui a, D Corolario 11. Las medianas hom´logas de dos tri´ngulos semejantes, estan o a en la misma raz´n que sus lados hom´logos. o o An tio Teorema 12 (Tercer criterio de semejanza:P-P-P). Si los tres lados de un tri´ngulo son respectivamente proporcionales a los a tres lados de otro tri´ngulo, entonces los dos tri´ngulos son semejantes. a a A’ D B Un iv ersi dad de A E C B’ Figura 16. C’
  • 18. 218 CAP´ ITULO 7. SEMEJANZA, JAIME ESCOBAR A. Demostraci´n. (Ver Figura 16.) Tomemos por hip´tesis que o o AB AC BC = = AB AC BC (∗) − → − → Por el axioma de construcci´n de segmento, existen D ∈ AB y E ∈ AC tales o que AD ∼ A B y AE ∼ A C , sustituyendo en (*), = = atic as AB AC = AD AE y por el corolario 3 (rec´ ıproco del corolario 2), ADE ∼ eM ABC, de esta semejanza se BC AB = AB DE o. d Por lo tanto, por el corolario 4, concluye que o sea que (∗∗), ept BC AB = AD DE atem DE||BC pero por hip´tesis o qui a, D BC AB = (∗ ∗ ∗) AB BC de (**) y (***) y por las propiedades de las fracciones: DE = B C o sea que An tio DE ∼ B C = de y por lo tanto, por el tercer criterio de congruencia de tri´ngulos L-L-L: a ∼ A B C y como ADE ∼ ABC, entonces por transitividad, ADE = ABC. dad ABC ∼ 7.4. Un iv ersi Corolario 12. Si las bases de dos tri´ngulos is´sceles son entre si como sus a o otros lados, entonces los tri´ngulos son semejantes. a ´ SEMEJANZA EN EL TRIANGULO ´ RECTANGULO Los resultados de aplicar los conceptos de semejanza al tri´ngulo rect´ngua a lo son de mucha importancia, pues obtendremos el teorema de Pit´goras y a aplicaciones al tri´ngulo y a los cuadril´teros, a las areas, etc. a a ´
  • 19. ´ ´ 7.4. SEMEJANZA EN EL TRIANGULO RECTANGULO 219 Definici´n 8. o a. La proyecci´n ortogonal de un punto exterior a una o recta, es el punto de intersecci´n de una recta perpendicular desde el o punto a la recta. b. La proyecci´n ortogonal de un segmento sobre una recta es el segmeno to determinado por las proyecciones ortogonales de los extremos del segmento sobre la recta. B atic as P A’ l B’ eM P’ atem A ¤ ept o. d Figura 17. An tio qui a, D En la Figura 17., la proyecci´n ortogonal del punto P sobre la recta l es o −→ − −→ − el punto P , ya que l ⊥ P P y {P } = l ∩ P P . La proyecci´n ortogonal del segmento AB sobre la recta l es el segmento o A B , donde A y B son las proyecciones ortogonales sobre l de A y B respectivamente. dad de Teorema 13 (Proporcionalidad en el tri´ngulo rect´ngulo). a a Si en un tri´ngulo rect´ngulo se traza la altura correspondiente a la a a hipotenusa, entonces: ersi a. Los dos nuevos tri´ngulos que resultan, son semejantes entre si y sea mejantes al tri´ngulo original. a Un iv b. La altura es media proporcional entre los segmentos que ella determina sobre la hipotenusa. c. Cada cateto es media proporcional entre la hipotenusa y la proyecci´n o del cateto sobre la hipotenusa. Demostraci´n. (Ver Figura 18.) o
  • 20. 220 CAP´ ITULO 7. SEMEJANZA, JAIME ESCOBAR A. C A B atic as D Figura 18. ADC ∼ ABC, eM atem a. Sabemos por el corolario 5, que si dos tri´ngulos rect´ngulos tienen un a a angulo agudo congruente, entonces los dos tri´ngulos son semejantes, ´ a por lo tanto CDB ∼ ABC ∼ CDB ept ADC ∼ o. d y por transitividad ABC An tio qui a, D b. Como ADC ∼ CDB y CAD ∼ DCB y ACD ∼ CBD entonces la = = relaci´n entre los lados hom´logos del ADC con los lados hom´logos o o o del CDB es AD AC DC ADC : = = CDB CD CB DB de luego CD2 = AD · DB o sea que CD es media proporcional entre AD y DB. ersi dad c. Como ADC ∼ ABC y ACD ∼ CBA y el angulo A es com´n, ´ u = entonces la relaci´n entre los lados hom´logos del ADC con los lados o o hom´logos del ABC es o Un iv ADC : ABC AD AC DC = = AC AB CB luego AC 2 = AD · AB o sea que AC es media proporcional entre AD y AB. Como CDB ∼ ABC, BCD ∼ CAB y el angulo B es com´n, se ´ u = demuestra en forma similar que CB 2 = AB · DB
  • 21. ´ ´ 7.4. SEMEJANZA EN EL TRIANGULO RECTANGULO 221 o sea que CB es media proporcional entre AB y DB. Teorema 14 (Teorema de Pit´goras). a El cuadrado de la medida de la hipotenusa en un tri´ngulo rect´ngulo es a a igual a la suma de los cuadrados de las medidas de los catetos. B eM D Figura 19. o. d A atem atic as C a, D ept Demostraci´n. (Ver Figura 19.) Sea ABC un tri´ngulo rect´ngulo en o a a C y sea CD la altura relativa a la hipotenusa, entonces por la parte c. del anterior teorema: CB 2 = AB · DB An tio y sumando estas dos expresiones, tenemos qui AC 2 = AD · AB, de AC 2 + CB 2 = AD · AB + AB · DB = AB(AD + DB) = AB · AB = AB 2 Un iv ersi dad Teorema 15 (Rec´ ıproco del teorema de Pit´goras). a Si en un tri´ngulo el cuadrado de la medida de un lado es igual a la suma a de los cuadrados de las medidas de los otros dos lados, entonces el tri´ngulo a es rect´ngulo. a Demostraci´n. (Ver Figura 20.) Sea el ABC tal que AC 2 = AB 2 + o BC 2 . Veamos que el ABC es rect´ngulo en B. Para ello, construyamos un a tri´ngulo A B C rect´ngulo en B , as´ : en una recta l fijo un punto B , a a ı por el axioma de construcci´n de segmento, existe un punto C en una de las o semirrectas determinadas por B en l, tal que B C ∼ BC; por el teorema de = la perpendicular por un punto de una recta, por B pasa m ⊥ l, por el axioma de construcci´n de segmento, existe un punto A en una de las semirrectas o
  • 22. CAP´ ITULO 7. SEMEJANZA, JAIME ESCOBAR A. 222 m A A’ l C as B’ C’ atic B atem Figura 20. ept A C 2 = A B 2 + B C 2. A B C es o. d eM determinadas por B en m, tal que B A ∼ BA, por lo tanto el = rect´ngulo en B . Por el teorema de Pit´goras a a ABC y AB = A B , BC = B C de AC = A C , A B C se tiene: An tio En los tri´ngulos a qui A C = AC a, D Pero por hip´tesis AC 2 = AB 2 + BC 2 , luego A C 2 = AC 2 y por tanto o dad luego, por el criterio L-L-L, se tiene que ersi ABC ∼ = ABC ABC es rect´ngulo a Un iv luego ABC ∼ A B C y como A B C es recto, entonces = en B. 7.5. APLICACIONES DEL TEOREMA DE ´ PITAGORAS Con los siguientes teoremas se demuestra la ley de cosenos en trigonometr´ ıa.
  • 23. ´ 7.5. APLICACIONES DEL TEOREMA DE PITAGORAS 223 Teorema 16 (Ley de cosenos). a. En un tri´ngulo obtus´ngulo, el cuadrado de la medida del lado opa a uesto al angulo obtuso es igual a la suma de los cuadrados de las ´ medidas de los otros dos lados, m´s el doble producto de la medida a de uno de estos lados por la proyecci´n del otro sobre ´l. o e atem atic as b. En un tri´ngulo cualquiera, el cuadrado de la medida del lado opuesto a al angulo agudo es igual a la suma de los cuadrados de las medidas ´ de los otros dos lados, menos el doble producto de la medida de uno de estos lados por la proyecci´n del otro sobre ´l. o e eM A B ept o. d A H1 H a, D C qui B An tio H1 (a) C H (b) dad de Figura 21. ersi Demostraci´n. a.) (Ver Figura 21.(a)) Supongamos que en el o ←→ ABC el o angulo ABC es obtuso y sea BH la proyecci´n de AB sobre BC y sea BH1 ´ Un iv ←→ la proyecci´n de BC sobre AB, por el Teorema ??, H − B − C y A − B − H1 ; o veamos que AC 2 = AB 2 + BC 2 + 2 · BC · BH y AC 2 = AB 2 + BC 2 + 2 · AB · BH1 Demostremos la primera expresi´n, la otra se hace en forma similar. o Por el teorema de Pit´goras en el AHB se tiene a AB 2 = AH 2 + HB 2 (∗)
  • 24. CAP´ ITULO 7. SEMEJANZA, JAIME ESCOBAR A. 224 Por el teorema de Pit´goras en el a AHC se tiene AC 2 = AH 2 + HC 2 (∗∗) restando (**) y (*): AC 2 −AB 2 = HC 2 −HB 2 (∗∗∗), pero como H −B −C, entonces HC = HB + BC y sustituyendo en (***) y despejando atic as AC 2 = AB 2 + (HB + BC)2 − HB 2 = AB 2 + HB 2 + BC 2 + 2 · HB · BC − HB 2 = AB 2 + BC 2 + 2 · BC · HB ABC el angulo ABC es ´ atem b.) (Ver Figura 21.(b)). Supongamos que en el ←→ agudo y sea BH la proyecci´n de AB sobre BC y sea BH1 la proyecci´n de o o eM ←→ BC sobre AB, por el Teorema ??, B − H − C y B − H1 − A; veamos que y AC 2 = AB 2 + BC 2 − 2 · AB · BH1 o. d AC 2 = AB 2 + BC 2 − 2 · BC · BH a, D ept Demostremos la primera expresi´n, la otra se hace en forma similar. o Por el teorema de Pit´goras en el AHB se tiene a (∗) qui AB 2 = AH 2 + HB 2 AHC se tiene An tio Por el teorema de Pit´goras en el a AC 2 = AH 2 + HC 2 (∗∗) dad de restando (**) y (*): AC 2 −AB 2 = HC 2 −HB 2 (∗∗∗), pero como B −H −C, entonces HC = BC − HB y sustituyendo en (***) y despejando Un iv ersi AC 2 = AB 2 + (BC − HB)2 − HB 2 = AB 2 + BC 2 + HB 2 − 2 · BC · HB − HB 2 = AB 2 + BC 2 − 2 · BC · HB Teorema 17 (Teorema de Stewart). En el ABC, D ∈ IntBC. Si BD = m, DC = n, AD = d, entonces d2 a = b2 m + c2 n − amn
  • 25. ´ 7.5. APLICACIONES DEL TEOREMA DE PITAGORAS 225 A c b d D m H n a C as B atem atic Figura 22. Demostraci´n. (Ver Figura 22.) Sea D ∈ BC en el o ←→ ABC, sea DH la a, D ept o. d eM proyecci´n de AD sobre BC; con el ADB pueden suceder tres casos: i. que o sea obtuso, ii. que sea recto, iii. que sea agudo. Mostremos el primer caso, los otros casos son similares. Como ADB es obtuso, entonces por el Teorema ?? B − D − H y ADC es agudo y BD + DC = BC; por el teorema anterior (ley de cosenos) en el ADB y en el ADC: (∗) AC 2 = AD2 + DC 2 − 2 · DC · DH (∗∗) An tio qui AB 2 = AD2 + BD2 + 2 · BD · DH multiplicando (*) por DC y (**) por BD y luego sumando: dad de AB 2 · DC + AC 2 · BD = AD 2 · (DC + BD) + BD 2 · DC + DC 2 · BD = AD2 · BC + BD · DC(DC + BD) = AD2 · BC + BD · DC · BC ersi luego AD2 · BC = AB 2 · DC + AC 2 · BD − BD · DC · BC es decir, Un iv d2 a = b2 m + c2 n − amn. Teorema 18. a.) La suma de los cuadrados de las medidas de dos lados de un tri´ngulo a es igual a dos veces el cuadrado de la medida de la mediana del tercer lado m´s la mitad del cuadrado de la medida del tercer lado. a b.) La diferencia de los cuadrados de las medidas de dos lados de un tri´ngulo a es igual a dos veces el producto de la medida del tercer lado por la proyecci´n o de la mediana correspondiente a este lado.
  • 26. 226 CAP´ ITULO 7. SEMEJANZA, JAIME ESCOBAR A. A c b ma M B H C as a atem atic Figura 23. eM Demostraci´n. (Ver Figura 23.) En el ABC, sea M el punto medio de o BC, ma la mediana relativa al lado BC y M H la proyecci´n de la mediana o ←→ qui a, D ept o. d AM = ma sobre BC, supongamos que AB > AC. Con el angulo AM B ´ pueden suceder tres casos: i. es obtuso, ii. es recto, iii. es agudo. Tomemos el caso i. y veamos que a.) c2 + b2 = 2 · m2 + 1 a2 a 2 b.) c2 − b2 = 2 · a · M H. En efecto, como AM B es obtuso entonces AM C es agudo, luego por el teorema de la ley de cosenos en el AM B y en el AM C: (∗) AC 2 = AM 2 + M C 2 − 2 · M C · M H (∗∗) de An tio AB 2 = AM 2 + BM 2 + 2 · BM · M H ersi dad sumando (*) y (**) y teniendo en cuenta que M es punto medio, o sea que M B = M C, entonces Un iv AB 2 + AC 2 = 2 · AM 2 + BM 2 + M C 2 BC 2 BC 2 = 2 · AM 2 + + 2 2 BC 2 1 = 2 · AM 2 + 2 = 2 · AM 2 + BC 2 2 2 1 2 c2 + b 2 = 2 · m 2 + · a a 2 restando (*) y (**) y teniendo en cuenta que M es punto medio, o sea que
  • 27. ´ 7.5. APLICACIONES DEL TEOREMA DE PITAGORAS 227 M B = M C,: AB 2 − AC 2 = 4 · M B · M H BC · M H = 2 · BC · M H =4 2 c2 − b 2 = 2 · a · M H qui A a, D ept o. d eM atem atic as Teorema 19 (Altura en funci´n de los lados). o En un ABC cuyos lados miden: BC = a, AC = b, AB = c; las alturas miden: 2 ha = p(p − a)(p − b)(p − c) a 2 p(p − a)(p − b)(p − c) hb = b 2 hc = p(p − a)(p − b)(p − c) c ımetro. donde p = a+b+c =semi-per´ 2 An tio c ha C ersi dad a de H B b Un iv Figura 24. Demostraci´n. (Ver Figura 24.) Sea ha = AH la altura relativa al lado o BC, con H pueden ocurrir los siguientes casos i. B − H − C, ii. B − C − H o H − B − C, iii. H ≡ B o H ≡ C. Mostremos el caso i. y supongamos que c > b (ver la Figura 24.), el caso c < b es similar, el caso c = b se deja como ejercicio; como los tri´ngulos a
  • 28. CAP´ ITULO 7. SEMEJANZA, JAIME ESCOBAR A. 228 AHB y AHC son rect´ngulos, entonces por el teorema de Pit´goras: a a c2 = h2 + BH 2 a 2 b = h2 + CH 2 a (7.1) (7.2) Como B − H − C entonces HC = a − BH, sustituyendo en 7.2 (7.3) as b2 = h2 + (a − BH)2 = h2 + a2 + BH 2 − 2aBH a a atic y por 7.1 en la expresi´n anterior o atem b2 = h2 + a2 + c2 − h2 − 2aBH = a2 + c2 − 2aBH a a a2 + c 2 − b 2 2a + despejando h2 a =c − a2 + c 2 − b 2 2a 2 = c2 − 4a2 c2 − (a2 + c2 − b2 )2 (a2 + c2 − b2 )2 = 4a2 4a2 de 2 dad h2 a 2 An tio c = h2 a qui 2 a, D y sustituyendo en 7.1 o. d a2 + c 2 − b 2 2a ept BH = eM como a = 0, ya que A, B, C son tres puntos distintos no colineales, despejando BH en la expresi´n anterior o (2ac + a2 + c2 − b2 )(2ac − a2 − c2 + b2 ) ((a + c)2 − b2 )(b2 − (a − c)2 ) = 4a2 4a2 (a + c + b)(a + c − b)(b + a − c)(b − a + c) = 4a2 (a + b + c)(a + c − b)(a + b − c)(b + c − a) = (7.4) 4a2 Un iv ersi = Como p = a+b+c 2 entonces a + b + c = 2p y tambi´n e p−a= a+b+c a + b + c − 2a b+c−a −a= = 2 2a 2
  • 29. ´ 7.5. APLICACIONES DEL TEOREMA DE PITAGORAS 229 por lo tanto b + c − a = 2(p − a) Similarmente a + b − c = 2(p − c) y a + c − b = 2(p − b), sustituyendo en 7.4 2p · 2(p − a) · 2(p − b) · 2(p − c) 4 = 2 · p · (p − a) · (p − b) · (p − c) 2 4a a ha = p · (p − a) · (p − b) · (p − c). ´ CONSTRUCCIONES BASICAS atem 7.5.1. 2 a as por lo tanto atic h2 = a eM 1. Dividir un segmento en n segmentos congruentes, con n entero positivo. X An o. d An−1 ept A2 C1 C2 Cn−1 Figura 25. B An tio qui A a, D A1 de Construcci´n. Para la construcci´n, haremos los siguientes pasos cono o secutivos (Ver Figura 25.). Un iv ersi dad −→ − • Por A trazo una semirrecta AX cualesquiera, tal que A, B y X sean tres puntos distintos no colineales. −→ − • Con centro en A y radio cualesquiera, trazo arco que corta a AX en A1 . −→ − • Con centro en A1 y el mismo radio, trazo arco que corta a AX en A2 de tal manera que A − A1 − A2 ; similarmente se hallan los puntos A3 , . . . , An−1 , An . • Uno An con B y por An−1 , An−2 , . . . , A2 , A1 trazo paralelas a An B las cuales cortan a AB en Cn−1 , Cn−2 , . . . , C2 , C1 . • AC1 ∼ C1 C2 ∼ · · · ∼ Cn−1 B = = =
  • 30. CAP´ ITULO 7. SEMEJANZA, JAIME ESCOBAR A. 230 Justificaci´n. Como o AA1 ∼ A1 A2 ∼ · · · ∼ An−1 An = = = y BAn ··· Cn−1 An−1 C 1 A1 atic AC1 ∼ C1 C2 ∼ · · · ∼ Cn−1 B = = = as entonces por el Teorema fundamental del paralelismo (Lema 1), eM atem 2. Dividir un segmento dado en una proporci´n dada p , donde p, q son o q enteros positivos. X o. d Q a, D B qui C Figura 26. An tio A ept P de Construcci´n. Para la construcci´n, haremos los siguientes pasos cono o secutivos (Ver Figura 26.). Un iv ersi dad −→ − • Por A trazo una semirrecta AX cualesquiera, tal que A, B y X sean tres puntos distintos no colineales. −→ − • Con centro en A y radio cualesquiera, trazo arco que corta a AX en A1 , este procedimiento lo efect´o p veces hasta completar un u segmento AP de longitud pAA1 , a continuaci´n de este segmento o y utilizando la misma medida AA1 construyo el segmento P Q de longitud qAA1 . • Uno Q con B y por P trazo paralela a QB la cual corta a AB en C. • el punto C es el punto pedido.
  • 31. ´ 7.5. APLICACIONES DEL TEOREMA DE PITAGORAS Justificaci´n. Como QB o de Tales en el tri´ngulo) a 231 P C, entonces por el Corolario 1 (Teorema CA pAA1 p = = CB qAA1 q 3. Hallar la cuarta proporcional de tres segmentos dados: a, b, c. atic B o. d Figura 27. X ept C eM atem P A as Y Q An tio qui a, D Construcci´n. Para la construcci´n, haremos los siguientes pasos cono o secutivos (Ver Figura 27.). −→ − • Trazo una semirrecta AX cualesquiera, − → • Trazo una semirrecta AY cualesquiera, que no este contenida en ←→ la recta AX Un iv ersi dad de − → • Con centro en A y radio a, trazo arco que corta a AY en P . − → • Con centro en P y radio b, trazo arco que corta a AY en Q, tal que A − P − Q. −→ − • Con centro en A y radio c, trazo arco que corta a AX en C. −→ − • Uno P con C y por Q trazo paralela a P C la cual corta a AX en B. • el segmento CB es el segmento pedido. Justificaci´n. Como P C o de Tales en el tri´ngulo) a QB, entonces por el Corolario 1 (Teorema AC AP = PQ CB o sea a c = b CB
  • 32. 232 CAP´ ITULO 7. SEMEJANZA, JAIME ESCOBAR A. 4. Dado C ∈ AB, hallar el conjugado arm´nico de C con respecto a AB. o P Q ¥ ¥ C D B atem atic as A eM Figura 28. o. d Construcci´n. Para la construcci´n, haremos los siguientes pasos cono o secutivos (Ver Figura 28.). ept • Trazo circunferencia de centro A y radio AC. a, D • Trazo circunferencia de centro B y radio BC. qui • En la circunferencia C(A, AC), trazo un radio cualesquiera AP no paralelo a AB. An tio • Por B trazo, en la circunferencia C(B, BC), el radio BQ tal que BQ AP ←→ • Uno P con Q y prolongo hasta cortar la recta AB en D . de • el punto D es el conjugado de C con respecto a AB. Un iv ersi dad Justificaci´n. Como BQ AP entonces ADP ∼ BDQ entonces, o teniendo en cuenta que AP y BQ son radios en las respectivas circunferencias, DA CA DA AP = o sea = BQ DB CB DB 5. Dado AB y dada la proporci´n p , donde p, q son enteros positivos. Hao q CA DA llar C, D conjugados arm´nicos de AB tal CB = DB = p . o q Construcci´n. Para la construcci´n, haremos los siguientes pasos cono o secutivos (Ver Figura 29.).
  • 33. ´ 7.5. APLICACIONES DEL TEOREMA DE PITAGORAS 233 X P Y Q ¦ ¦ C A B D as R atic Z atem Figura 29. An tio qui a, D ept o. d eM ←→ −→ − • Trazo una semirrecta cualquiera AX que no este contenida en AB. −→ − −→ −→ − − • En el mismo semiplano, trazo la semirrecta BY tal que BY AX − → −→ − y trazo tambi´n la semirrecta BZ opuesta a la semirrecta BY . e −→ − • Sobre la semirrecta AX y con la misma unidad de medida α, trazo el segmento AP tal que AP = p · α. −→ − • Sobre la semirrecta BY y con la misma unidad de medida α, trazo el segmento BQ tal que BQ = q · α. − → • Sobre la semirrecta BZ y con la misma unidad de medida α, trazo el segmento BR tal que BR = q · α. ←→ • Uno P con Q y prolongo hasta cortar la recta AB en D . de • Uno P con R el cual corta a AB en C . AP D ∼ entonces, y tambi´n e Y Z entonces Un iv Justificaci´n. Como AX o ersi dad • Los puntos C y D son conjugados arm´nicos con respecto a AB o bajo la raz´n p . o q BQD y AP C ∼ BRC DA AP = DB BQ o sea DA p·α p = = DB q·α q CA AP = CB BR o sea CA p·α p = = CB q·α q
  • 34. CAP´ ITULO 7. SEMEJANZA, JAIME ESCOBAR A. 234 luego DA CA = CB DB 6. Hallar la media proporcional de dos segmentos a y b dados. X atem atic as D § C eM O B Figura 30. l o. d A qui • Sobre una recta l fijo un punto A. a, D ept Construcci´n. Para la construcci´n, haremos los siguientes pasos cono o secutivos (Ver Figura 30.). • Con centro en A y radio a trazo arco que corta a l en B. de An tio • Con centro en B y radio b trazo arco que corta a l en C, tal que A − B − C. −→ − • Por B trazo BX ⊥ l. dad • Hallo O punto medio de AC. ersi • Trazo semicircunferencia de centro O y radio OA, la cual corta a −→ − BX en D. Un iv • El segmento BD es media proporcional entre a y b. Justificaci´n. Como AC es di´metro, entonces ACD es rect´ngulo y o a a como DB es altura relativa a la hipotenusa en dicho tri´ngulo, entonces, a por el Teorema 13 (Proporcionalidad en el tri´ngulo rect´ngulo) a a BD2 = BA · BC = a · b es decir BD es media proporcional entre a y b.
  • 35. 7.6. APLIC. DE LA SEMEJANZA A LA CIRCUNFERENCIA 7.6. 235 APLICACIONES DE LA SEMEJANZA A LA CIRCUNFERENCIA as Teorema 20 (Teorema de Tolomeo). En un cuadril´tero c´ a ıclico, el producto de las medidas de las diagonales es igual a la suma de los productos de las medidas de los lados opuestos. atic A atem d c eM a D E B o. d C An tio qui Figura 31. a, D F ept b Demostraci´n. (Ver Figura 31.) Por el axioma de construcci´n de angulo, o o ´ − → existe una semirrecta AF ⊂ ←→ con F sobre la circunferencia, tal que AB: C DAC ∼ BAF , sea {E} = AF ∩ DB y como CAF ∼ CAF entonces por el = = axioma de suma (o resta) de angulos congruentes, DAF ∼ BAC ´ = ∼ BAF y DCA ∼ ABE (por En los ADC y AEB se tiene: DAC = = Teorema del angulo inscrito), entonces por el criterio A-A: ´ Un iv ersi dad de π ADC ∼ luego ADC : AEB AEB AD AC DC = = AE AB EB luego AC · EB = DC · AB (∗)
  • 36. 236 CAP´ ITULO 7. SEMEJANZA, JAIME ESCOBAR A. En los DAE y ABC se tiene: DAE ∼ BAC y ADE ∼ ACB (por = = Teorema del angulo inscrito), entonces por el criterio A-A: ´ DAE ∼ ABC luego DA DE AE = = AC BC AB DAE : ABC (∗∗) atic DA · BC = DE · AC as luego atem sumando (*) y (**): eM DC · AB + DA · BC = AC · EB + DE · AC = AC(EB + DE) = AC · BD AC · BD = a · c + b · d. o. d es decir, An tio qui a, D ept Teorema 21. Si dos cuerdas se interceptan en el interior de una circunferencia entonces el producto de las medidas de los segmentos determinados por el punto de intersecci´n en una de las cuerdas es igual al producto de las medidas de o los segmentos determinados en la otra cuerda. C O ersi Un iv D ¨ X dad de A B Figura 32. Demostraci´n. (Ver Figura 32.) Sean AB y CD cuerdas tales que {X} = o AB ∩ CD y A − X − B y C − X − D. En los AXC y BXD se tiene
  • 37. 7.6. APLIC. DE LA SEMEJANZA A LA CIRCUNFERENCIA 237 que: por opuestos por el v´rtice AXC ∼ BXD y por el Teorema del angulo e ´ = ∼ XDB, luego por el criterio A-A, inscrito CAX = AXC ∼ = BXD luego XA AC XC = = XD BD XB as AXC : BXD o. d eM atem atic o sea que XA · XB = XC · XD Nota: 1.) Obs´rvese que si por ejemplo el punto X ≡ A ≡ C, es decir, los e dos segmentos se cortan sobre la circunferencia, entonces tambi´n se cumple e que XA · XB = XC · XD = 0. 2.) El resultado de este teorema nos muestra que para cualquier cuerda que pase por el punto X se cumple que XA · XB permanece constante o sea que este producto no depende de la cuerda, sino del punto X. a, D ept El siguiente teorema se deja como ejercicio, es el rec´ ıproco del teorema anterior. de An tio qui Teorema 22. Si dos segmentos se interceptan en un punto que esta en el interior de los dos segmentos y el producto de las medidas de los segmentos determinados por el punto de intersecci´n en el primer segmento es igual al producto de las o medidas de los segmentos determinados por el punto en el segundo segmento, entonces los extremos de los segmentos est´n sobre una circunferencia. a Un iv ersi dad Teorema 23. Si desde un punto X exterior a una circunferencia se trazan dos semirrectas secantes l y m que cortan a la circunferencia en A, B y C, D respectivamente, entonces XA · XB = XC · XD Demostraci´n. (Ver Figura 33.) Por el Teorema del angulo inscrito BAD ∼ o ´ = BCD y el X es com´n para los XAD y XBC entonces por el criterio u A-A XAD ∼ XBC
  • 38. CAP´ ITULO 7. SEMEJANZA, JAIME ESCOBAR A. 238 A B © O C D as X luego eM XA XD AD = = XC XB BC luego ept XA · XB = XC · XD o. d XAD : XBC atem atic Figura 33. qui a, D Nota: El resultado de este teorema nos muestra que para cualquier semirrecta que pase por el punto X se cumple que XA · XB permanece constante o sea que este producto no depende de la semirrecta, sino del punto X. An tio El rec´ ıproco del anterior teorema tambi´n es cierto, se deja como ejercicio. e Teorema 24 (Rec´ ıproco). dad de Si desde un punto X se trazan dos semirrectas l y m y A, B son puntos de l y C, D son puntos de m, tales que ersi XA · XB = XC · XD, Un iv entonces los puntos A, B, C, D est´n sobre una circunferencia. a Teorema 25. Si desde un punto exterior a una circunferencia se trazan dos semirrectas, una tangente y la otra secante, entonces el segmento entre el punto y el punto de tangencia es media proporcional entre los segmentos determinados entre el punto exterior y los puntos de intersecci´n de la secante con la o circunferencia.
  • 39. 7.7. EJE RADICAL Y SUS PROPIEDADES 239 A B O M X as C atem atic Figura 34. Demostraci´n. (Ver Figura 34.) Como por el teorema del angulo semio ´ XAC ∼ eM ept XBC, a, D XAC : XBC XBC, en- XA XC AC = = XC XB BC qui luego XAC y o. d 1 inscrito el BCX = 2 CM B y X es com´n para los u tonces por el criterio A-A, luego EJE RADICAL Y SUS PROPIEDADES de 7.7. An tio XA · XB = XC · XC = XC 2 . Un iv ersi dad Definici´n 9 (Potencia de un punto con respecto a una circunfeo rencia). La potencia de un punto X con respecto a una circunferencia C(O, r) es el producto XA · XB, donde A y B son los puntos de intersecci´n o de la circunferencia con una recta que pasa por X. Notaci´n: la potencia del punto X con respecto a la circunferencia o C(O, r) se denota por pX;O , es decir, pX;O = XA · XB Nota a.) De acuerdo a los teoremas 21 y 23, todas las rectas que pasan por el punto X tienen igual potencia, por lo tanto, la potencia depende solamente
  • 40. 240 CAP´ ITULO 7. SEMEJANZA, JAIME ESCOBAR A. del punto y la circunferencia. b.) Si X es un punto exterior a la C(O, r) y d es la distancia del punto X al centro O de la circunferencia, entonces (ver la Figura 35.) pX;O = XA · XB = (XO + OA)(XO − OB) = (d + r)(d − r) = d2 − r2 ←→ O B X a, D qui Figura 35. ept o. d eM A atem atic as donde A, B son los puntos de intersecci´n de la recta XO con la C(O, r). o En este caso pX;O 0, ya que d r de An tio c.) Con el punto X y la circunferencia C(O, r) pueden suceder tres casos: 1. X ∈ ExtC(O, r), en este caso vimos que pX;O 0, ya que d r 2. X ∈ IntC(O, r), en este caso pX;O = d2 − r2 0, ya que d r 3. X ∈ C(O, r), en este caso pX;O = d2 − r2 = 0, ya que d = r ersi dad En resumen, la potencia es positiva en el exterior de la circunferencia, negativa en el interior de la circunferencia y es cero cuando el punto esta sobre la circunferencia. Un iv d.) Si X ≡ O, entonces d = 0 y por tanto pX;O = d2 − r2 = −r2 , este es el valor m´ ınimo de la potencia, ya que d = 0 es el valor m´ ınimo de d. e.) Por el teorema 25, la potencia de un punto exterior a una circunferencia es igual al cuadrado de la medida del segmento tangente desde el punto X a la circunferencia C(O, r), es decir, pX;O = XT 2 , donde T es el punto de tangencia.
  • 41. 7.7. EJE RADICAL Y SUS PROPIEDADES 241 f.) La potencia de un punto interior a una circunferencia es igual y negativa, del cuadrado de la semi-cuerda perpendicular al di´metro que pasa por el a punto. C as O B atic X atem A eM D ept o. d Figura 36. An tio qui a, D (Ver Figura 36.) En efecto, sea AB di´metro y X ∈ AB y sea CD una a cuerda tal que CD ∩ AB = {X} y AB⊥CD, por tanto X es punto medio de CD, entonces CD 2 2 de pX;O = −XA · XB = −XC · XD = −XC 2 = −XD2 = − Un iv ersi dad Teorema 26 (Teorema del eje radical). El lugar geom´trico de los puntos de igual potencia con respecto a dos e circunferencias no conc´ntricas, es una recta perpendicular a la recta que e pasa por los centros. Demostraci´n. (Ver Figura 37.) Sean las circunferencias C(O, r) y C(O , r ), o sea M el punto medio de OO y sea X un punto tal que pX;O = pX;O (∗), sea ←→ H la proyecci´n de X sobre OO , veamos que cualquiera que sea el punto o ←→ X con la propiedad (*), tendr´ como proyecci´n sobre OO el punto H. a o En efecto, por la hip´tesis, por la propiedad b.) hecha en la nota anterior o
  • 42. CAP´ ITULO 7. SEMEJANZA, JAIME ESCOBAR A. 242 X H M O’ atic as O eM atem Figura 37. o. d y por el Teorema 18 b), se tiene pX;O = pX;O (XO) − (r)2 = (XO )2 − (r )2 por la propiedad b) (XO)2 − (XO )2 = (r)2 − (r )2 2 · OO · M H = (r)2 − (r )2 por el Teorema 18 b) (r)2 − (r )2 luego M H = 2 · OO An tio qui a, D ept 2 de como r, r , OO son constantes y OO = 0, entonces M H es constante y como M es fijo entonces H es fijo, cualquiera sea el punto X, por lo tanto los puntos X que cumplen con la propiedad (*) est´n sobre una recta perpendicular a a dad ←→ OO . Un iv ersi La recta cuya existencia esta garantizada por el anterior teorema, le damos el siguiente nombre: Definici´n 10 (Eje Radical). La recta cuyos puntos tienen igual potencia o con respecto a dos circunferencias, se le llama Eje Radical. Propiedades del Eje Radical. 1. Si las dos circunferencias se interceptan, entonces el eje radical pasa por los puntos de intersecci´n, ya que cada punto de intersecci´n tiene o o potencia igual a cero con respecto a las dos circunferencias.
  • 43. 7.7. EJE RADICAL Y SUS PROPIEDADES 243 2. Si las dos circunferencias son tangentes, entonces el eje radical es la tangente com´n a ambas circunferencias, ya que la potencia en el punto u de tangencia es cero con respecto a las dos circunferencias y la tangente com´n es perpendicular a la recta que pasa por los centros de las dos u circunferencias. as 3. Si las dos circunferencias son conc´ntricas y distintas, entonces no hay e 2 2 2 eje radical, ya que d − r = (d ) − (r )2 atic Teorema 27 (Propiedades del Eje Radical). atem a.) Las tangentes desde un punto del Eje Radical a las dos circunferencias, son congruentes. ept o. d eM b.) Los Ejes Radicales de tres circunferencias, cuyos centros son no colineales, tomados de dos en dos, son concurrentes, (este punto de concurrencia se le llama Centro Radical). An tio qui a, D Demostraci´n. a.) (ver Figura 38.) Sea X un punto del Eje Radical y sean o XT y XT1 tangentes a las circunferencias C(O, r) y C(O , r ) en T y T1 2 respectivamente, entonces por el Teorema 25 pX;O = XT 2 y pX;O = XT1 y como X pertenece al Eje Radical, entonces pX;O = pX;O luego 2 XT 2 = XT1 de luego XT = XT1 , o sea que XT ∼ XT1 = b.)(ver Figura 39.) Sea l el Eje Radical de C(O, r) y C(O , r ) y sea l el ←→ ←→ ersi dad Eje Radical de C(O , r ) y C(O , r ) por lo tanto l ⊥OO y l ⊥O O . Como O, O , O son no colineales, entonces l y l se interceptan, sea Un iv {X} = l ∩ l . Veamos que X ∈ l . En efecto, como X ∈ l entonces pX;O = pX;O y como X ∈ l entonces pX;O = pX;O (∗) (∗∗)
  • 44. 244 CAP´ ITULO 7. SEMEJANZA, JAIME ESCOBAR A. X T T1 H O atic as O’ eM atem Figura 38. o. d l O’ ept l An tio qui X a, D O l de O” ersi entonces de (*) y (**) dad Figura 39. luego X ∈ l . Un iv pX;O = pX;O Observaci´n. o De la parte b.) del teorema anterior se concluye que: 1. Si las tres circunferencias son secantes dos a dos, entonces las cuerdas comunes son concurrentes. 2. Si las tres circunferencias son tangentes dos a dos, entonces las tangentes
  • 45. 7.7. EJE RADICAL Y SUS PROPIEDADES 245 comunes son concurrentes. Con el Eje Radical se pueden hacer construcciones de circunferencias. Ejemplo. Construir una circunferencia que pase por dos puntos y sea tangente a una recta dada. Demos el problema por construido. Supongamos que los puntos dados son ←→ as A, B y la recta dada es l, se presentan dos situaciones: a) AB ∩l = ∅, ←→ atic ←→ b) AB ∩l = ∅. ←→ atem a) Si AB ∩l = ∅, sea {X} =AB ∩l = ∅, sea C(O, r) la circunferencia buscada y sea C(O , r ) una circunferencia cualesquiera que pase por A y B, ←→ a, D ept o. d eM entonces AB es el Eje Radical de estas dos circunferencias, por lo tanto las tangentes desde el punto X a las dos circunferencias son congruentes; si XT es la tangente a la C(O , r ) y XT es la tangente a la circunferencia buscada C(O, r), entonces XT = XT . O’ An tio B T1 Un iv X T ersi dad de O T’ A qui m Figura 40. Construcci´n. Para la construcci´n, haremos los siguientes pasos consecuo o tivos (Ver Figura 40.). Uno A con B y prolongo hasta cortar l en X.
  • 46. CAP´ ITULO 7. SEMEJANZA, JAIME ESCOBAR A. 246 Trazo m la mediatriz de AB. Por un punto cualesquiera O de m, trazo una circunferencia que pase por A, B. Desde X trazo XT tangente a la circunferencia de centro O Con centro en X y radio XT trazo arcos que cortan a l en T y T1 . ←→ atic as Las circunferencias que pasan por A, B, T y por A, B, T1 son las circunferencias pedidas (dos soluciones). ←→ ←→ atem b) Si AB ∩l = ∅, luego AB l. Sea C(O, r) la circunferencia buscada y sea T el punto de tangencia entre la C(O, r) y l, por lo tanto OT ⊥ l, pero como ←→ ←→ eM AB l, entonces OT ⊥ AB, luego OT es mediatriz de AB ept o. d Construcci´n. Para la construcci´n, haremos los siguientes pasos consecuo o tivos. a, D Uno A con B . Trazo m la mediatriz de AB que corta a l en T . An tio qui Trazo circunferencia que pasa por A, B, T , que es la circunferencia pedida. ersi dad de Ejemplo. Construir una circunferencia que pase por dos puntos y sea tangente exteriormente a una circunferencia dada. Demos el problema por construido. Supongamos que los puntos dados son A, B y la circunferencia dada es C(O , r ) y sea m la mediatriz de AB, se presentan dos casos: a) O ∈ m, b) O ∈ m / Un iv a) O ∈ m, sea C(O , r ) una circunferencia que pase por A, B e inter/ ←→ cepte a la circunferencia dada C(O , r ) en los puntos C, D, por lo tanto CD es el Eje Radical de estas dos circunferencias, como la circunferencia buscada C(O, r) y la circunferencia dada C(O , r ) son tangentes, entonces la tangente l com´n a estas dos circunferencias es el Eje Radical de ambas y u ←→ como O ∈ m, entonces l y CD se interceptan en un punto X; como los Ejes / Radicales de tres circunferencias cuyos centros no son colineales son concurrentes, entonces X es el centro radical de las tres circunferencias, luego las
  • 47. 7.7. EJE RADICAL Y SUS PROPIEDADES 247 m A O atem D C atic X T l as O” B O’ eM ept o. d T1 Figura 41. a, D tangentes desde X a las tres circunferencias son congruentes. qui Construcci´n. Para la construcci´n, haremos los siguientes pasos consecuo o tivos (Ver Figura 41.). An tio Uno A con B . Trazo m la mediatriz de AB . dad de Por un punto O de m trazo circunferencia que pasa por A, B y que corte a la circunferencia dada C(O , r ) en los puntos C, D. ←→ ersi Uno C con D y prolongo hasta cortar AB en X. Un iv Desde X trazo trazo XT y XT1 tangentes a la circunferencia dada C(O , r ). Las circunferencias que pasan por A, B, T y A, B, T1 son las circunferencias pedidas (dos soluciones). b) Si O ∈ m. Sea {T } = m ∩ C(O , r ), en este caso, O, T, O son colineales y por tanto T es el punto de tangencia.
  • 48. 248 CAP´ ITULO 7. SEMEJANZA, JAIME ESCOBAR A. Construcci´n. Para la construcci´n, haremos los siguientes pasos consecuo o tivos. Uno A con B . Trazo m la mediatriz de AB, la cual intercepta a la circunferencia dada C(O , r ) en T . Un iv ersi dad de An tio qui a, D ept o. d eM atem atic as Trazo circunferencia que pase por los puntos A, B, T y esta es la circunferencia pedida.
  • 49. 7.8. EJERCICIOS Y PROBLEMAS DE SEMEJANZA 7.8. 249 Ejercicios y Problemas de Semejanza 1. Sea ∆ABC un tri´ngulo inscrito en la circunferencia C(O, r), sea AD, a con D ∈ C(O, r), la bisectriz del BAC y sea {E} = BC ∩ AD. Mostrar que a) BD 2 = AD · ED, b) ∆BED ∼ ∆AEC B atem atic as 2. En la figura, si ABD ∼ = DBE ∼ EBC, entonces = AD = AB·BD . EC BE·BC A C E B N K a, D M D C qui H 3. Si ABCD es un paralelogramo y M N AB, AB = 12, DM = 4, DE = 6, KB = 2KH. Hallar: a) AM , b) DH, c) DC, d) KF , d) LM , e) M N . ept A o. d eM D An tio E dad de 4. Sea ABC un tri´ngulo cualesquiera, por el v´rtice A trazamos una a e −→ − semirrecta AX paralela al lado BC. Desde M punto medio de BC −→ − se traza una recta cualesquiera que corta a AX en N , AC en P y la prolongaci´n de AB en Q. Probar que o Un iv ersi QN PN = PM QM 5. Demostrar que el cuadrado de la medida de la bisectriz AE de un angulo exterior de un ∆ABC es igual al producto de las medidas de ´ los segmentos que la bisectriz determina sobre la recta que contiene al lado opuesto, menos el producto de las medidas de los otros dos lados. (Ayuda: siendo C(O, r) la circunferencia que circunscribe al tri´ngulo a ←→ y {D} = C(O, r)∩ AE, observar los ∆DAC y ∆ABE).
  • 50. 250 CAP´ ITULO 7. SEMEJANZA, JAIME ESCOBAR A. 6. Se tiene un cuadrado ABCD de lado a. Se traza una circunferencia que pasa por el v´rtice A y por los puntos medios de los lados AB y AD. e Probar que la medida de una tangente a dicha circunferencia trazada desde el punto C es igual a a. 7. Construir un tri´ngulo dadas las raz´n entre los lados c y b (es decir, a o p c dado b = q ), la mediana ma y el lado a ( c = p , ma , a) b q atem atic as 8. Por un punto D del lado AB de un ABC se traza DE AC (E sobre BC), de tal manera que DB = e, CE = 2e, BE = 2AD. Calcular los lados AB y BC del tri´ngulo. a eM 9. Demostrar que en un mismo tri´ngulo las alturas son inversamente a proporcionales a sus respectivos lados. ept o. d a 10. Considere la C(O, r). Sea AB un di´metro. Se traza por B una tangente y por A una secante cualesquiera que corta a la C(O, r) en M y a la tangente en N . Probar que AM.AN = 4r 2 . qui a, D 11. Sea la C(O, r) y AB di´metro y sea M un punto en la prolongaci´n a o de AB, se trazan las tangentes M N y M P a la C(O, r), la cuerda N P corta al di´metro AB en C. Demostrar que a An tio MA CA = CB MB Un iv ersi dad de 12. Sea C(O, r), se traza una cuerda CD, O el punto medio de CD, se traza la circunferencia de centro O y di´metro CD, sea AB di´metro a a de C(O, r) perpendicular a CD; se trazan AT y AT tangentes a la C(O ), la cuerda T T corta a AB en F . Demostrar que O es punto medio de BF . 13. En ABC rect´ngulo en A la hipotenusa mide a y la altura relativa a a la hipotenusa mide h, se inscribe un cuadrado con un lado sobre la hipotenusa. Calcular el lado del cuadrado en t´rminos de a y h. e 14. En una circunferencia de di´metro 40cm. , hallar la medida de la mayor a y la menor cuerda que puede trazarse por un punto situado a 12cm. del centro. Explicar porque es la mayor y la menor.
  • 51. 7.8. EJERCICIOS Y PROBLEMAS DE SEMEJANZA 251 15. Desde el punto medio D del lado AB del ABC, rect´ngulo en A, se a o traza DE ⊥ BC, con E ∈ BC. Demostrar la relaci´n EC 2 − EB 2 = AC 2 ←→ atic CD, demuestre que atem cia que circunscribe al tri´ngulo y F ∈ C(O, r) a ADC ∼ F BC). as 16. Demostrar que el cuadrado de la bisectriz de un angulo exterior de un ´ tri´ngulo es igual al producto de los segmentos que la bisectriz determia na en el lado opuesto menos el producto de los otros dos lados (Ayuda: si CD es la bisectriz exterior en el ABC y C(O, r) es la circunferen- eM 17. En un ABC is´sceles con AB = AC, se traza CD ⊥ AB. Demostrar o la relaci´n o o. d AB 2 + BC 2 + CA2 = BD2 + 2DA2 + 3CD2 An tio qui a, D ept 18. Si el tri´ngulo del ejercicio anterior fuera un tri´ngulo equil´tero, mostrar a a a que las suma de los cuadrados de las medidas de los lados es igual a cuatro veces el cuadrado de la medida de la altura. − − → 19. El ABC esta inscrito en una C(O, r), sea AD la bisectriz de A con − − → D ∈ C(O, r) y sea E ∈ BC ∩ AD. Mostrar que: a) BD2 = AD.ED, b) BED ∼ AEC. ←→ ←→ 20. LM N T es un paralelogramo, LT = 15, LM = 8, RN = 12, N R⊥LR, ←→ ←→ de T H⊥M N , H ∈ M N . Hallar T H. ←→ ←→ ←→ ABC, sea AN BC y M punto medio de BC, sea P ∈N M ←→ ←→ dad 21. Dado el ersi ∩ AB y Q ∈N M ∩ AC . Demostrar que Un iv PN QN = PM QM 22. Dado un ABC is´sceles con CA ∼ CB y la circunferencia tangente o = a los lados congruentes en A y B. Desde un punto M del arco de la circunferencia en el interior del tri´ngulo, se traza M D ⊥ AB, M F ⊥ a CB y M E ⊥ CA. Mostrar que M D2 = M E.M F
  • 52. 252 CAP´ ITULO 7. SEMEJANZA, JAIME ESCOBAR A. 23. Sean AA , BB , CC las alturas de un en el punto H. Demostrar que: ABC; estas alturas se cortan AA .A H = A C.A B, BB .B H = B A.B C, CC .C H = C B.C A 24. Se d´ una circunferencia de centro O y di´metro AB, por un punto M a a sobre la prolongaci´n de AB, se trazan las tangentes M N y M P a la o circunferencia, la cuerda N P corta al di´metro en C. Demostrar que: a atic as CA MA = CB MB eM atem 25. Demostrar que si dos tri´ngulos tienen sus lados respectivamente paraa lelos o respectivamente perpendiculares, entonces dichos tri´ngulos son a semejantes. o. d 26. Dado un paralelogramo ABCD, tal que: DC = 32, AD = 17, AC = 28. Hallar DB. AB = a, D ept 27. Sea ∆ABC con CE, BD, AF bisectrices. Si CA = 32, 20, CB = 36. Hallar AE, CF, AD. An tio qui 28. Demostrar que la suma de las longitudes de los catetos de un tri´ngulo a rect´ngulo, no excede la longitud de la diagonal de un cuadrado consa truido sobre la hipotenusa del tri´ngulo como lado. a de 29. Demostrar que en un paralelogramo la suma de los cuadrados de los lados es igual a la suma de los cuadrados de las diagonales. ersi dad 30. Sea un tri´ngulo rect´ngulo ABC (recto en A), donde: AB = 8, AC = a a 15 . Calcular BC, la altura AH y los segmentos BH y HC. Se traza por B una paralela a AC que corta la altura AH en I. Evaluar AH, HI y BI. Un iv 31. Sobre el lado AB de un angulo BAC, se toman dos puntos D y E y ´ por esos puntos se trazan dos paralelas que cortan al lado AC en F y G respectivamente; se trazan F E y por el punto G, una paralela a F E que corta a AB en H. Demostrar que AE 2 = AD.AH. 32. Dado un cuadril´tero ABCD, sea O el punto de intersecci´n de sus a o diagonales. Por el punto O se traza una paralela a BC que corta a AB en E; luego se traza por O una paralela a CD que corta a AD en F .
  • 53. 7.8. EJERCICIOS Y PROBLEMAS DE SEMEJANZA AE a. Mostrar que AB = una misma raz´n). o b. Mostrar que EF AF AD 253 (comparar cada una de estas razones con BD. c. Se traza OG AB y cortando BC en G y OH en H. Mostrar que CG.DH = BG.CH. AD, corta a DC atic as 33. Demostrar que las paralelas a los lados de un tri´ngulo ABC, trazadas a por el punto G de concurrencia de las medianas, dividen cada lado en tres partes iguales. ←→ ←→ atem 34. Sea ABCD un cuadril´tero, sea F sobre AC y E sobre DB tales que a F B||DC y EC||AB. Mostrar que AD||F E. o. d eM 35. El per´ ımetro de un tri´ngulo mide 90 cm.. Sabiendo que las medidas a de los lados est´n en la relaci´n 1 : 2 : 3. Calcular la medida de cada a o lado. a, D ept 36. Demuestre que en tri´ngulos semejantes las alturas hom´logas, las mea o dianas hom´logas y las bisectrices hom´logas son proporcionales a los o o lados hom´logos. o A dad de An tio qui 37. En la figura, la C(O, x) esta inscrita en el sector circular ABC. Si m(ABC) = 60o , hallar x en funci´n de r. o r (Rta.: 3 ) . x O B C r Un iv ersi 38. Si en un tri´ngulo rect´ngulo, X y Y son las medidas de los catetos y a a Z es la medida de la altura correspondiente a la hipotenusa, demuestre que: 1 1 1 + 2 = 2 X2 Y Z a a 39. Los catetos AB y AC de un tri´ngulo rect´ngulo ∆ABC miden respectivamente 4a y 3a. Por el punto medio M de AB se traza hacia el exterior del tri´ngulo, un segmento M N perpendicular a AB e igual a a su mitad. Hallar la medida de N C.
  • 54. CAP´ ITULO 7. SEMEJANZA, JAIME ESCOBAR A. 254 40. Los lados de un tri´ngulo miden 10, 12 y 18. Si el per´ a ımetro de un tri´ngulo semejante a ´l mide 1,200, cuales son las medidas de los lados a e del segundo tri´ngulo? Cu´nto miden las tres alturas, las tres medianas a a y las tres bisectrices del √ primer tri´ngulo? √a √ (Rta.: 300, 360, 540, 30 41, 30 176, 30 209) A B F E C m(BEC) = m(BF C) + m(BDC) atem D atic as 41. Si ABCD es un rect´ngulo de laa dos a y 3a. Demostrar que eM 42. a1 , b1 , c1 son puntos medios de los lados del tri´ngulo ∆ABC. Dea muestre: ∆ABC ∼ ∆a1 b1 c1 ∼ ∆Ac1 b1 ∼ ∆Bc1 a1 ∼ ∆Cb1 a1 ept o. d 43. ABCD es un paralelogramo O ∈ AC, OX ⊥ AD, OY ⊥ AB. DeAB mostrar que OX = AD OY 44. Dos circunferencias son tangentes interiormente en el punto A. Del ←→ ←→ qui a, D punto A, se trazan las secantes AC y AE. B y D pertenecen a la circunferencia interior. C y E pertenecen a la circunferencia exterior. Demuestre que ∆ABD ∼ ∆ACE. An tio 45. Sea AB un di´metro en la C(O, r), por B se traza una tangente a la a circunferencia y por A se traza una secante cualquiera que intercepta la circunferencia en M y a la tangente en N . Demostrar que dad de AM · AN = 4r 2 Un iv ersi 46. Demostrar que en un trapecio el segmento paralelo a las bases que pasa por el punto de intersecci´n de las diagonales, es bisecado por o dicho punto. 47. Dos tri´ngulos rect´ngulos son semejantes. Si los catetos hom´logos a a o miden a y a , b y b y las hipotenusas hom´logas miden c y c , demostrar o que aa + bb = cc . 48. Sean AB y CD dos cuerdas perpendiculares de una circunferencia de radio r y sea {X} = AB ∩ CD. Demostar que XA2 + XB 2 + XC 2 + XD2 = 4r2
  • 55. 7.8. EJERCICIOS Y PROBLEMAS DE SEMEJANZA 255 49. Las bases mayor y menor de un trapecio miden a y b respectivamente. Por un punto de uno de los lados no paralelos se traza un segmento paralelo a las bases. El segmento divide al lado en la relaci´n m : n. o Calcular la longitud del segmento. 50. Dado el ←→ ←→ ABC, se consideran los puntos D, E, F sobre las rectas ←→ ←→ ←→ ←→ atem atic as BC, AC, AB respectivamente. Si las rectas AD, BE y CF pasan por el centro O de la circunferencia circunscrita del ABC, cuyo radio es R, mostrar que 1 1 2 1 + + = AD BE CF R eM 51. En un tri´ngulo el punto de concurrencia de: las alturas, el de las a medianas y el de las mediatrices est´n alineados (Recta de Euler ). a o. d 52. Demostrar que en todo tri´ngulo, la bisectriz se encuentra entre la a mediana y la altura trazadas desde el mismo v´rtice. e a, D ept 53. Las bases de un trapecio miden 20 y 12 y los lados no paralelos miden 10 y 12. Calcular la medida de las diagonales y de las alturas y los lados del tri´ngulo que se forma al prolongar los lados no paralelos. a An tio qui 54. ABCD es un cuadril´tero. AB = a, BC = b, CD = c, DA = d, CE = a EA = m, BF = F D = n, EF = r. Demuestre: a2 + b2 + c2 + d2 = (2m)2 + (2n)2 + 4r2 . dad de 55. Dados dos segmentos de longitud a cm. y b cm., construir con regla y comp´s: a a) un segmento de longitud ab cm. b) un segmento de longitud a cm. b ersi 56. Trazar las tangentes exteriores y las interiores a dos circunferencias. Un iv 57. Constru´ un tri´ngulo ABC, conociendo ır a a) BC, ABC y BN que es la altura desde B, (a, β, hb ). b) BC, AM y AH que son la mediana y la altura correspondientes a BC, (a, ma , ha ). c) BC, y la altura y la bisectriz BH y CD, (a, hb , vc ). d ) BC y las alturas BH y CP , (a, hb , hc ).
  • 56. CAP´ ITULO 7. SEMEJANZA, JAIME ESCOBAR A. 256 e) BC, AC y la altura BH, (a, b, hb ). f ) BC, BAC y la mediana AM , (a, α, ma ). g) BC, BAC y la altura BH, (a, α, hb ). h) Los pies de las tres medianas. i ) Las tres medianas: ma , mb , mc . as j ) ABC, ACB y el per´ ımetro , (β, γ, p; donde p = a + b + c). atem atic 58. Construir un tri´ngulo equil´tero, conociendo el radio de la circunfea a rencia inscrita. eM 59. Construir un tri´ngulo equil´tero, conociendo su per´ a a ımetro. o. d 60. Construir un tri´ngulo is´sceles conociendo el per´ a o ımetro y la medida de la altura correspondiente a la base. a) AB a, D ept 61. Construir una circunferencia que pase por dos puntos A y B y que sea tangente a una recta l; con A y B del mismo lado con respecto a l. l, qui b) AB ∩ l = {P }. An tio 62. Construir una circunferencia que sea tangente a dos rectas paralelas dadas y que pase por un punto dado. dad de 63. Construir una circunferencia que sea tangente a dos rectas que se cortan y pase por un punto en el interior del angulo entre las dos rectas. ´ ersi 64. Construir una circunferencia que sea tangente a una circunferencia y a una recta dadas y que pase por un punto dado. Un iv 65. Dado un punto en el interior de una circunferencia, construir una cuerda tal que el punto dado sea punto medio de dicha cuerda. 66. Sea AB di´metro de una circunferencia, A, B, M colineales con B entre a A y M , M N tangente en N y N C ⊥ AB, C entre A y B. Mostrar que MA CA = CB MB
  • 57. 7.8. EJERCICIOS Y PROBLEMAS DE SEMEJANZA 257 67. Dado un angulo XOY y un punto A en el interior de XOY , trazar por ´ −→ − − − → A una recta que corte a OX en M y a OY en N , de tal forma que A sea punto medio de M N . 68. Dos circunferencias de centros O y O1 y de radios diferentes son secantes en A. Trazar por A una cuerda BC, de tal forma que A sea el punto medio de BC. (B ∈ C(O) y C ∈ C(O1 ) ). atic as 69. Constru´ un tri´ngulo conociendo dos angulos y la suma de las medidas ır a ´ de dos de sus lados. atem 70. Construir un rect´ngulo ABCD conociendo AB y el angulo AOB fora ´ mado por las diagonales. o. d eM 71. Construir un tri´ngulo ABC, rect´ngulo en A, conociendo la suma de a a las medidas de los catetos y el angulo C. ´ ept 72. Construir un rect´ngulo conociendo su per´ a ımetro y su diagonal. a, D 73. Construir un trapecio conociendo sus bases y sus diagonales. 74. Construir un cuadril´tero conociendo sus lados y una de sus diagonales. a An tio qui 75. Construir un cuadril´tero inscriptible conociendo BD, y AC que son a sus diagonales, el angulo A y el lado AB. ´ de 76. Circunscribir un tri´ngulo equil´tero en una circunferencia de radio a a dado. dad 77. Construir una circunferencia que sea tangente a dos rectas dadas y cuyo centro est´ sobre una recta dada. e ersi 78. Construir una circunferencia tangente a tres rectas dadas. Un iv 79. Trazar una recta tangente a una circunferencia dada y paralela a una recta dada. 80. Construir un tri´ngulo conociendo: a a) Los pies E, F, D de las tres alturas. b) Un lado BC, el angulo opuesto α , y la suma o la diferencia de ´ los otros dos lados (a, α, c − b), (a, α, c + b).
  • 58. 258 CAP´ ITULO 7. SEMEJANZA, JAIME ESCOBAR A. c) Un angulo β y las alturas opuestas AD y CF . (β, ha , hc ). ´ d ) Un angulo β , la altura BE y la altura AD, (β, hb , ha ). ´ ´ e) Un lado BC, un angulo β , y la mediana AD (a, β, ma ). f ) El per´ ımetro, un angulo y la altura bajada desde el v´rtice del ´ e angulo: (p, α, ha ). ´ as g) La altura y bisectriz bajadas del mismo v´rtice y el radio de la e circunferencia inscrita (vc , hc , r). atem atic h) La altura y la mediana bajadas desde el mismo v´rtice y el radio e de la circunferencia circunscrita (ma , ha , R). 81. Construir un tri´ngulo conociendo: a o. d eM a) Dos lados y la longitud de la bisectriz del angulo comprendido ´ (a, c, vb ). ept b) La base AB , el angulo opuesto y la suma de las medidas de los ´ lados que comprenden este angulo (c, γ, a + b). ´ Un iv ersi dad de An tio qui a, D 82. Por un punto P exterior a una circunferencia trazar una secante P AB, PA tal que AB = m donde m, n son dos n´meros naturales dados. u n